NURS320 MedSurg Final Exam Review: Gas Exchange, Perfusion, and Blood Administration

Pataasin ang iyong marka sa homework at exams ngayon gamit ang Quizwiz!

A nurse is caring for a client who has pneumonia. Assessment findings include temperature 37.8C (100F), respirations 30/min, blood pressure 130/76, heart rate 100/min, and SaO2 on 91% room air. Prioritize the following nursing interventions. - Administer antibiotics - Instruct the client to obtain a yearly influenza vaccination - Administer oxygen therapy - Perform a sputum culture

1. Administer oxygen therapy 2. Perform a sputum culture 3. Administer antibiotics 4. Instruct the client to obtain a yearly influenza vaccination

A nurse is caring for a client who asks why the provider prescribed a daily aspirin? Which of the following responses should the nurse make? A. "Aspirin reduces the formation of blood clots that could cause a heart attack." B. "Aspirin relieves the pain due to myocardial ischemia." C. "Aspirin dissolves the clots that are forming in your coronary arteries." D. "Aspirin relieves headaches that are caused by other medications."

A. "Aspirin reduces the formation of blood clots that could cause a heart attack."

A nurse is teaching a group of clients about influenza. Which of the following client statements indicates an understanding of the teaching? A. "I should wash my hands after blowing my nose to prevent spreading the virus." B. "I need to avoid drinking fluids if I develop symptoms." C. "I need a flu shot every 2 years because of the different flu strains." D. "I should cover my mouth with my hand when I sneeze."

A. "I should wash my hands after blowing my nose to prevent spreading the virus."

A nurse is caring for a group of clients. Which of the following clients are at risk for a pulmonary embolism? (Select all that apply.) A. A client who has a BMI of 30 B. A female client who is postmenopausal C. A client who has a fractured femur D. A client who is a marathon runner E. A client who has chronic atrial fibrillation

A. A client who has a BMI of 30 C. A client who has a fractured femur E. A client who has chronic atrial fibrillation

A nurse is assessing a client who has pulmonary edema related to heart failure. Which of the following findings indicates effective treatment of the client's condition? A. Absence of adventitious breath sounds B. Presence of a nonproductive cough C. Decrease in respiratory rate at rest D. SaO2 86% on room air

A. Absence of adventitious breath sounds Rationale: Adventitious breath sounds occur when there is fluid in the lungs. The absence of adventitious breath sounds indicates that the pulmonary edema is resolving.

Monitoring of which of the following biophysiological parameters occurs during a sleep study? (Select all that apply.) A. Cardiac rhythm B. Electroencephalogram C. Eye movement D. Blood pressure E. Oxygen saturation

A. Cardiac rhythm B. Electroencephalogram C. Eye movement E. Oxygen saturation

A nurse is caring for a client following a thoracentesis. Which of the following manifestations should the nurse recognize as risks for complications? (Select all that apply.) A. Dyspnea B. Localized bloody drainage on the dressing C. Fever D. Hypotension E. Report of pain at puncture site

A. Dyspnea C. Fever D. Hypotension

A nurse is caring for a client who is receiving vecuronium during mechanical ventilation. Which of the following medications should the nurse anticipate administering with this medication? (Select all that apply.) A. Fentanyl B. Furosemide C. Midazolam D. Famotidine E. Dexamethasone

A. Fentanyl C. Midazolam

A nurse is providing discharge teaching for a client who has a prescription for furosemide 40 mg PO daily. The nurse should instruct the client to take this medication at which of the following times of day? A. Morning B. Immediately after lunch C. Immediately before dinner D. Bedtime

A. Morning

The nurse understands that a/an _______ is used in the evaluation of cardiogenic shock. A. PA catheter B. LVAD C. RVAD D. IABP

A. PA catheter

A client who is anticipating total hip replacement is considering autologous transfusion. When teaching this client about autologous transfusion, it is important to emphasize that... A. it reduces the risk of mismatched blood. B. a hemoglobin level above 9.5 mg/dL is required to receive this transfusion. C. there is no need to test the blood for infectious diseases. D. donations can be made every other day.

A. it reduces the risk of mismatched blood. Rationale: Mismatched blood can cause an immune response to another person's antigens. Because the client is their own donor in an autologous transfusion, there is no risk of exposure to another person's antigens. A hemoglobin level of at least 11 g/dL is required for autologous donation.

Which assessment findings are considered modifiable risk factors for cardiac disease? (Select all that apply.) A. obesity B. smoking C. parent dies of heart attack at 55 years of age D. diabetes mellitus E. female 60 years of age

A. obesity B. smoking D. diabetes mellitus

A nurse is preparing a blood transfusion for a client who has type A blood. The nurse should know that the client can safely receive blood from blood group O because... A. type O blood contains no A antigens. B. type A blood contains O antibodies. C. type O blood contains no A antibodies. D. type A blood contains O antigens.

A. type O blood contains no A antigens. Rationale: Type O blood contains no antigens at all, which is why clients who have type O blood are considered universal donors. Their blood can be transfused to anyone who has any ABO-related blood type without putting them at risk for an ABO incompatibility. It is the specific antigens in the transfused blood that can trigger hemolytic reactions. Because type O blood has no antigens, it is safe for this client and for any other client.

Which patient scheduled for a MRI with contrast requires immediate action by the nurse? A. 18 year old who has a suspected muscle tear in the right thigh muscle and in severe pain is scheduled for MRI of leg B. 30 year old with chronic renal failure and lower back pain is scheduled for MRI of the spine C. 40 year old with a history of migraines that are getting worse is scheduled for MRI of the head D. 75 year old with right side and leg pain and has difficulty walking is scheduled for MRI of the hip

B. 30 year old with chronic renal failure and lower back pain is scheduled for MRI of the spine

What is the priority treatment for a patient with cystic fibrosis? A. Nutritional counseling B. Airway clearance C. Medication administration D. Patient education

B. Airway clearance

A nurse is caring for four clients. Which of the following clients should the nurse identify as being at risk fo developing rheumatic endocarditis? A. Older adult who has COPD B. Child who has streptococcal pharyngitis C. Middle-aged adult who has lupus erythematosus D. Young adult who recently received a body tattoo

B. Child who has streptococcal pharyngitis

A nurse in the ED is assessing a client who has sustained multiple rib fractures and has a flail chest. Which of the following findings should the nurse expect? (Select all that apply.) A. Bradycardia B. Cyanosis C. Hypotension D. Dyspnea E. Paradoxical chest movement

B. Cyanosis C. Hypotension D. Dyspnea E. Paradoxical chest movement

A nurse is caring for a newly admitted client who has emphysema. The nurse should place the client in which of the following positions to promote effective breathing? A. Lateral position with a pillow at the back and over the chest to support the arm B. High-Fowler's position with the arms supported on the overbed table C. Semi-Fowler's position with pillows supporting both arms D. Supine position with the head of the bed elevated to 15°

B. High-Fowler's position with the arms supported on the overbed table Rationale: The nurse should place the client in a position that allows for greater expansion of the chest, such as sitting upright and leaning slightly forward while supporting both arms with pillows for comfort on the overbed table. The semi-Fowler's, supine, and lateral positions do not promote maximum chest expansion to facilitate breathing.

A nurse is caring for a client who is receiving warfarin for anticoagulation therapy. Which of the following laboratory test results indicates to the nurse that the client needs an increase in the dosage? A. aPTT 38 seconds B. INR 1.1 C. PT 22 seconds D. D-dimer negative

B. INR 1.1

After oxygen has been administered, the next priority intervention the nurse would initiate for a patient with a pulmonary embolus is the administration of which of these therapies? A. Normal saline IV fluid B. IV heparin C. Platelet administration D. Antibiotics for inflammatory fever

B. IV heparin

It is determined that Carlos now needs to undergo a cardiac catheterization with possible percutaneous transluminal coronary angioplasty. Which action is most appropriate by the nurse when Carlos returns back to the floor? A. Obtain lung sounds and apical pulse every 15 minutes B. Instruct Carlos to stay in bed and keep leg immobilized C. Limit one visitor per hour and monitor Carlos ' temperature D. Check the gag reflex before starting fluids

B. Instruct Carlos to stay in bed and keep leg immobilized

The nurse is reviewing orders for a newly admitted patient with PAD in the right lower extremity. The nurse should follow up with the provider about which order? A. Begin Plavix 75 mg PO daily B. Keep affected extremity elevated C. Begin Lisinopril 10 mg PO daily D. Encourage light exercise as tolerated

B. Keep affected extremity elevated

A nurse is caring for a client who is being treated for heart failure and has a prescription for furosemide. The nurse should plan to monitor for which of the following adverse effects of the medication? A. Shortness of breath B. Lightheadedness C. Dry cough D. Metallic taste

B. Lightheadedness Rationale: Furosemide can cause a substantial drop in blood pressure, resulting in lightheadedness or dizziness.

Which of the following indicates an abnormal assessment finding of the respiratory system? A. Symmetrical chest expansion B. Muscle retractions with inhalation C. The sound of resonance with percussion of lung tissue D. Pink mucosa of the mouth and nares

B. Muscle retractions with inhalation

A nurse is assisting the provider to care for a client who has developed a spontaneous pneumothorax. Which of the following actions should the nurse perform first? A. Assess the client's pain B. Obtain a large-bore IV needle for decompression C. Administer lorazepam D. Prepare for chest tube insertion

B. Obtain a large-bore IV needle for decompression

Approximately 10 minutes after a client returns from surgery with a tracheostomy tube, the nurse assesses increased noisy respiratory rate, restlessness and increased pulse. What action should the nurse take immediately? A. Take the blood pressure B. Suction the tracheostomy tube C. Drain water from the O2 tube D. Change the tracheostomy tube

B. Suction the tracheostomy tube Rationale: There are definite signs of an obstruction of air flow, suctioning the airway is the best way to relieve this.

The nurse understands adequate teaching has been done by which of the following statements? A. "I got a flu shot last year; I'm covered for a while." B. "I don't need a flu shot; I never get sick." C. "I guess it's important to get a flu shot every year." D. "I don't get a flu shot, but I make sure my kids get one."

C. "I guess it's important to get a flu shot every year."

A nurse is reviewing prescriptions for a client who has acute dyspnea and diaphoresis. The client states, "I am anxious and unable to get enough air." Vital signs are HR 117, RR 38, temp 38.4C (101.2F), and BP 100/54. Which of the following nursing actions is the priority? A. Notify the provider B. Administer heparin via IV infusion C. Administer oxygen therapy D. Obtain a CT scan

C. Administer oxygen therapy

What assessment would the nurse identify as a hallmark finding of left-sided heart failure? A. Ascites B. Bradycardia C. Crackles D. Edema

C. Crackles

Pulse oximetry may not be a reliable indicator of oxygen saturation in the patient... A. With a fever B. Who is anesthetized C. In hypovolemic shock D. Receiving high concentrations of oxygen

C. In hypovolemic shock Rationale: The patient in hypovolemic shock has decreased BP and perfusion, which will impact the pulse ox reading.

A patient with asthma presents with which symptoms? A. Cough, elevated blood pressure B. Decreased respirations, fatigue C. Increased respirations, wheezes D. Increased sputum, decreased respirations

C. Increased respirations, wheezes

Which is true of the electrical conduction system of the heart? A. It is primarily controlled by the movement of uncharged ions. B. It has a positive resting membrane potential. C. It is reflected in the waveforms on the electrocardiogram. D. It requires cells that respond only to a stimulus from the autonomic nervous system.

C. It is reflected in the waveforms on the electrocardiogram.

Which of the following is included in an arterial blood gas analysis? A. End-tidal CO2 B. Hemoglobin C. PaCO2 D. Sodium

C. PaCO2

Which order would the nurse question in the initial management of acute pericarditis? A. Aspirin B. Colchicine C. Prednisone D. NSAIDs

C. Prednisone

Which of the following structures actively participates in gas exchange? A. Bronchi B. Visceral pleura C. Respiratory bronchioles D. Terminal bronchioles

C. Respiratory bronchioles

The nurse is caring for a patient with seasonal allergy-related rhinitis. The nurse correlates which of the following symptoms with rhinitis? A. Productive cough B. Earache C. Rhinorrhea D. Headache

C. Rhinorrhea

A nurse is assessing a client who has bacterial pneumonia. Which of the following manifestations should the nurse expect? A. Decreased fremitus B. SaO2 95% on room air C. Temperature 38.8° C (101.8° F) D. Bradypnea

C. Temperature 38.8° C (101.8° F) Rationale: Increased fremitus, an oxygen saturation lower than 95%, and tachypnea are expected findings for bacterial pneumonia.

Which blood level is essential that the nurse monitor when Eleanor is taking Solumedrol? A. white blood cells B. potassium level C. glucose level D. sodium level

C. glucose level Rationale: Solumedrol is a steroid, so it has the potential to raise her blood glucose level.

The nurse is aware that the greatest risk factor for laryngeal cancer is... A. age. B. alcohol use. C. tobacco use. D. occupational hazards.

C. tobacco use.

When a patient with vitamin B12 deficiency is counseled about diet, what statement by the patient indicates an understanding of the cause of the anemia? A. "I know I need to eat more fruits and vegetables." B. "I have cut out all fried foods in my diet." C. "I have been eating more organic foods." D. "I have been having beef or fish at least once a day."

D. "I have been having beef or fish at least once a day."

A nurse is instructing a patient on the use of an incentive spirometer. Which of the following statements by the client indicates understanding of the teaching? A. "I will place the adapter on my finger to read my blood oxygen saturation level." B. "I will lie on my back with my knees bent." C. "I will rest my hand over my abdomen to create resistance." D. "I will take in a deep breath and hold it before exhaling."

D. "I will take in a deep breath and hold it before exhaling."

A nurse is caring for a client with a pulmonary embolism. Which of the following interventions is the nurse's priority? A. Provide a quiet environment. B. Encourage use of incentive spirometry every 1 to 2 hr. C. Obtain a blood sample for electrolyte study. D. Administer heparin via continuous IV infusion.

D. Administer heparin via continuous IV infusion. Rationale: When using the airway, breathing, circulation approach to client care, the nurse should place priority on stabilizing circulation to the lungs by administering heparin to prevent further clot formation. Therefore, this is the priority intervention.

A nurse is admitting a client who has suspected rheumatic endocarditis. The nurse should expect a prescription for which of the following laboratory tests to assist in confirmation of this diagnosis? A. ABGs B. Serum albumin C. Liver enzymes D. Throat culture

D. Throat culture

A nurse is caring for a client who has endocarditis. Which of the following findings should the nurse recognize as a potential complication? A. Ventricular depolarization B. Guillain-Barré syndrome C. Myelodysplastic syndrome D. Valvular disease

D. Valvular disease Rationale: Valvular disease or damage often occurs as a result of inflammation or infection of the endocardium.

A nurse is caring for a client who is postoperative and has a respiratory rate of 9/min secondary to general anesthesia effects and incisional pain. Which of the following ABG values indicates the client is experiencing respiratory acidosis? A. pH 7.50, PO2 95 mm Hg, PaCO2 25 mm Hg, HCO3- 22 mEq/L B. pH 7.50, PO2 87 mm Hg, PaCO2 35 mm Hg, HCO3- 30 mEq/L C. pH 7.30, PO2 90 mm Hg, PaCO2 35 mm Hg, HCO3- 20 mEq/L D. pH 7.30, PO2 80 mm Hg, PaCO2 55 mm Hg, HCO3- 22 mEq/L

D. pH 7.30, PO2 80 mm Hg, PaCO2 55 mm Hg, HCO3- 22 mEq/L

A nurse is teaching a client who has a new prescription for clopidogrel. Which of the following instructions should the nurse include? (Select all that apply.) A. "Avoid taking herbal supplements while taking this medication." B. "Monitor for the presence of black, tarry stools." C. "Take this medication when you have pain." D. "Schedule a weekly PT test." E. "Limit food sources containing vitamin K while taking this medication."

A. "Avoid taking herbal supplements while taking this medication." B. "Monitor for the presence of black, tarry stools."

Which blood type allele is considered the recessive gene? A. O B. A C. B D. All are dominant

A. O

A nurse is assessing a client following a gunshot wound to the chest. For which of the following findings should the nurse monitor to detect a pneumothorax? (Select all that apply.) A. Tachypnea B. Deviation of the trachea C. Bradycardia D. Decreased use of accessory muscles E. Pleuritic pain

A. Tachypnea B. Deviation of the trachea E. Pleuritic pain

The charge nurse is monitoring the care of several critically ill patients in the ICU. Which patient requires immediate intervention by the provider? A. The patient with a PA catheter remaining in the wedge position B. The patient with an SvO2 of 55% C. The patient with an SVR of 1,300 D. The patient with a CO of 3.2

A. The patient with a PA catheter remaining in the wedge position

The pulmonary edema associated with ARDS is caused by... A. increased permeability of the ACM. B. right ventricular failure with pulmonary hypertension. C. left ventricular failure due to poor oxygenation. D. fluid overload related to resuscitation in the first phase.

A. increased permeability of the ACM.

Which patient is at greatest risk for developing iron-deficiency anemia (IDA)? A. A 6-year-old African American boy with no health problems B. A 15-year-old African American pregnant female C. A 52-year-old Mexican American female with hypertension D. A 72-year-old Caucasian male with cardiac disease

B. A 15-year-old African American pregnant female

The nurse understands that the priority intervention with the patient with TB is which of the following? A. Antibiotic administration B. Initiation of isolation C. TB test D. Chest x-ray

B. Initiation of isolation

A nurse is assessing a client who has a pulmonary embolism. Which of the following manifestations should the nurse expect? (Select all that apply.) A. Bradypnea B. Pleural friction rub C. Hypertension D. Petechiae E. Tachycardia

B. Pleural friction rub D. Petechiae E. Tachycardia

A patient is a female age 35, who smokes a pack of cigarettes a day, has a total cholesterol of 99 and is obese. Her living father had a heart attack at the age of 52 years. This female patient has how many modifiable risk factors? A. One B. Two C. Three D. Four

B. Two Rationale: Smoking and obesity are modifiable. Total cholesterol is also a modifiable risk factor, BUT the patient's cholesterol level is normal.

Parents are inquiring about the blood type of their son. Mom is type O and dad is type AB. Which blood type may the child possess? (Select all that apply. Hint: find answer via a Punnet Square.) A. Type O B. Type A C. Type B D. Type AB

B. Type A C. Type B

Carlos is admitted to the ER. Which question will be most appropriate for the nurse to ask in order to determine if Carlos is experiencing angina or myocardial infarction? A. "Why did the pain start?" B. "How did you get to the hospital?" C. "Do you have any other symptoms?" D. "Did you follow your diet and exercise?"

C. "Do you have any other symptoms?"

A nurse is performing the immediate postoperative assessment of a patient who has just undergone CEA. What is the most important assessment to be reported immediately? A. A complaint of 7/10 pain B. Falling back to sleep after assessment C. An asymmetric smile D. Complaint of a sore throat

C. An asymmetric smile

A nurse in an emergency department is caring for a client who has a blood pressure of 254/139 mm Hg. The nurse recognizes that the client is in a hypertensive crisis. Which of the following actions should the nurse take first? A. Initiate seizure precautions. B. Tell the client to report vision changes. C. Elevate the head of the client's bed. D. Start a peripheral IV.

C. Elevate the head of the client's bed. Rationale: The greatest risk to this client is organ injury due to severe hypertension. Therefore, the first action the nurse should take is to elevate the head of the client's bed to reduce blood pressure and promote oxygenation.

A nurse is admitting a client who has a leg ulcer and a history of diabetes mellitus. Which of the following focused assessments should the nurse use to help differentiate between an arterial ulcer and a venous stasis ulcer? A. Explore the client's family history of peripheral vascular disease. B. Note the presence or absence of pain at the ulcer site. C. Inquire about the presence or absence of claudication. D. Ask if the client has had a recent infection.

C. Inquire about the presence or absence of claudication. Rationale: Knowing if the client is experiencing claudication helps differentiate venous from arterial ulcers. Clients who have arterial ulcers experience claudication, but those who have venous ulcers do not.

A nurse providing care for a patient whose BP readings are consistently 130/85 mm Hg should anticipate which medical plan of care? (Select all that apply.) A. Diagnostic testing for TOD B. Initiation of diuretics C. Modifications of diet and exercise D. Echocardiogram E. A stress test

C. Modifications of diet and exercise

A nurse is caring for a client who has asthma and is receiving albuterol. For which of the following adverse effects should the nurse monitor the client? A. Hyperkalemia B. Dyspnea C. Tachycardia D. Candidiasis

C. Tachycardia Rationale: The nurse should monitor the client for tachycardia, which is a common adverse effect of this medication, especially if the client uses albuterol on a regular basis. Additionally, the nurse should monitor the client for hypokalemia, which is a potential adverse effect of albuterol, and for a decrease in dyspnea, as this is a therapeutic effect of albuterol.

It appears Carlos may be having an myocardial infarction. Which test would best confirm the this diagnosis? A. Homocysteine B. CK-BB C. Troponin D. LDL

C. Troponin

A nurse is planning care for a client who has asthma. Which of the following medications should the nurse plan to administer during an acute asthma attack? A. Cromolyn sodium B. Prednisone C. Fluticasone/salmeterol D. Albuterol

D. Albuterol Rationale:

A nurse is caring for a client 2 hr after admission. The client has an SaO2 of 91%, exhibits audible wheezes, and is using accessory muscles when breathing. Which of the following class of medications should the nurse expect to administer? A. Antibiotic B. Beta-blocker C. Antiviral D. Beta2 agonist

D. Beta2 agonist Rationale: Administer a beta2 agonist, which causes dilation of the bronchioles to relieve wheezing and open the airways.

A nurse is assessing a client who has a history of DVT and is receiving warfarin. Which of the following findings should indicate to the nurse that the medication is effective? A. Hemoglobin 14 g/dL B. Minimal bruising of extremities C. Decreased blood pressure D. INR 2.0

D. INR 2.0 Rationale: The nurse should identify that an INR of 2.0 is within the desired reference range of 2.0 to 3.0 for a client who has a deep-vein thrombosis and is receiving warfarin to reduce the risk of new clot formation and a stroke.

A nurse is preparing a client for coronary angiography. Which of the following findings should the nurse report to the provider prior to the procedure? A. Hemoglobin 14.4 g/dL B. History of peripheral arterial disease C. Urine output 200 mL/4 hr D. Previous allergic reaction to shellfish

D. Previous allergic reaction to shellfish Rationale: The contrast medium used for coronary angiography is iodine-based. Clients who have a history of allergic reaction to shellfish often react to iodine and might need a steroid or antihistamine prior to the procedure.

A nurse is teaching a client who has heart failure and new prescriptions for furosemide and digoxin. Which of the following information should the nurse include? (Select all that apply.) A. Weight daily, first thing each morning B. Decrease intake of potassium C. Expect muscle weakness while taking digoxin D. Hold digoxin if HR is less than 70 bpm E. Decrease sodium intake

A. Weight daily, first thing each morning E. Decrease sodium intake

The nurse correlates which diagnostic result as increasing the risk for infection in the patient with leukemia? A. WBC 113/mm3 B. ANC 500 mm3 C. Hbg 8.6 g/dL D. Platelets 112,000 mm3

B. ANC 500 mm3

The nurse hears a loud systolic murmur at the second intercostal space right sternal border. What valve problem is the patient likely experiencing? A. Aortic regurgitation B. Aortic stenosis C. Pulmonic regurgitation D. Pulmonic stenosis

B. Aortic stenosis

A nurse is caring for a client who has a chest tube and drainage system in place. The nurse observes that the chest tube was accidentally removed. Which of the following actions should the nurse take first? A. Obtain a chest x-ray B. Apply sterile gauze to the insertion site C. Place tape around the insertion site D. Assess respiratory status

B. Apply sterile gauze to the insertion site

A nurse is teaching a client who has a new prescription for an ACE inhibitor to treat hypertension. The nurse should instruct the client to notify their provider if they experience which of the following adverse effects of this medication? A. Tendon pain B. Persistent cough C. Frequent urination D. Constipation

B. Persistent cough Rationale: A persistent cough is an adverse effect of ACE inhibitors. The client should report this finding to the provider and discontinue the medication.

When administering a transfusion of packed red blood cells, it is important to... A. allow the blood to warm to room temperature for 1 hr. B. make sure the entire unit is transfused within 4 hr. C. begin the blood transfusion at a rate of 10 mL/hr. D. change the blood tubing after every unit infused.

B. make sure the entire unit is transfused within 4 hr. Rationale: Infusion times that exceed 4 hr increase the risk for bacterial proliferation. Ideally, a unit of packed RBCs is infused within 2 hr. Clients who are at risk for fluid-volume excess will require slower rates of infusion; however, the entire transfusion must not exceed 4 hr.

A nurse is caring for a client who has a chest tube following a lobectomy. Which of the following items should the nurse keep easily accessible for the client? A. Extra drainage system B. Suture removal set C. Container of sterile water D. Nonadherent pads

C. Container of sterile water Rationale: The nurse should have a container of sterile water in a location that is easily accessible for this client. The nurse should plan to place the open end of the tubing into the sterile water if the tubing becomes disconnected to prevent a pneumothorax.

A nurse is caring for a client who had an onset of chest pain 24 hr ago. The nurse should identify that an increase in which of the following values is diagnostic of a myocardial infarction? A. Myoglobin B. C-reactive protein C. Creatine kinase-MB D. Homocysteine

C. Creatine kinase-MB Rationale: Creatine kinase-MB is the isoenzyme specific to the myocardium. Elevated creatine kinase-MB indicates myocardial muscle injury. Myoglobin is elevated following an MI, and with skeletal muscle injury. However, it is not specific to the cardiac muscle.

Which of the following actions should a nurse take prior to starting a blood transfusion? A. Establish intravenous access with a 22-gauge needle. B. Prime an infusion set with lactated Ringer's solution. C. Ensure that informed consent has been obtained from the client. D. Suggest that the client consider autologous transfusion.

C. Ensure that informed consent has been obtained from the client. Rationale: It is the responsibility of the prescribing health care provider to answer the client's questions about the need, risks, and benefits of a procedure. A nurse can witness the client's signature indicating informed consent. This must be done prior to obtaining or administering the blood. It is best to use an 18- or 19-gauge needle for blood transfusion. Blood is always administered with 0.9% sodium chloride solution, and never with a solution that contains dextrose, because it causes lysis of red blood cells, or with lactated Ringer's.

The nurse is reviewing the laboratory results of her patient and notes that a cardiac troponin level was drawn. This test was drawn to determine which diagnosis? A. Atrial fibrillation B. Ventricular tachycardia C. Myocardial infarction D. Congestive heart failure

C. Myocardial infarction

A nurse is completing the admission physical assessment of a client who has a mitral valve insufficiency. Which of the following findings should the nurse expect? A. S4 heart sound B. Petechiae C. Neck vein distention D. Splenomegaly

C. Neck vein distention

A nurse is caring for a client who is admitted to the ED with a BP of 266/147 mm Hg. The client reports a headache and double vision. The client states, "I ran out of diltiazem 3 days ago, and I am unable to purchase more." Which of the following actions should the nurse take first? A. Administer acetaminophen for headache B. Provide teaching regarding the importance of not abruptly stopping an antihypertensive C. Obtain IV access and prepare to administer an IV antihypertensive D. Call social services for a referral for financial assistance in obtaining prescribed medication

C. Obtain IV access and prepare to administer an IV antihypertensive

What is a probable initial assessment finding for a patient with a low hemoglobin count? A. Increased and bounding peripheral pulses B. Hypertension C. Pallor and fatigue D. Moist mucus membranes

C. Pallor and fatigue

What information should be included in the teaching plan for a 76-year-old patient after a physical? A. Initiating a new strenuous exercise regimen is recommended. B. Limit physical activity and exercise. C. Report any new or excessive fatigue. D. Excessive fatigue is not unusual as you age.

C. Report any new or excessive fatigue.

A nurse is assessing a client who has acute respiratory distress syndrome (ARDS). Which of the following findings should the nurse report to the provider? A. Decreased bowel sounds B. Oxygen saturation 92% C. CO2 24 mEq/L D. Intercostal retractions

D. Intercostal retractions Rationale: The nurse should report intercostal retractions to the provider because this finding indicates increasing respiratory compromise in a client who has ARDS. The nurse should identify that decreased bowel sounds and the given values for oxygen saturation and carbon dioxide are within the expected findings for a client who has ARDS.

Which condition requires the nurse to educate the patient on steps to prevent vasoconstriction? A. Fluid Volume Deficit B. Beurger's Disease C. Hypernatremia D. Raynaud's Disease

D. Raynaud's Disease Rationale: Raynaud's disease is characterized by vasoconstriction in the digits. Beurger's disease is an arterial disorder that is directly associated with smoking.

The nurse questions which order for the patient with cardiomyopathy? A. ACE inhibitors B. Beta blockers C. Diuretics D. SL nitroglycerin

D. SL nitroglycerin

A nurse is assisting a provider who is performing a thoracentesis at the bedside of a client. Which of the following actions should the nurse take? (Select all that apply.) A. Wear goggles and a mask during the procedure. B. Cleanse the procedure area with an antiseptic solution. C. Instruct the client to take deep breaths during the procedure. D. Position the client laterally on the affected side before the procedure. E. Apply pressure to the site after the procedure.

A. Wear goggles and a mask during the procedure. B. Cleanse the procedure area with an antiseptic solution. E. Apply pressure to the site after the procedure. Rationale: The nurse and provider should both wear goggles and a mask to reduce the risk for exposure to pleural fluid. The use of an antiseptic solution decreases the risk for infection, which is increased due to the invasive nature of the procedure. The application of pressure decreases the risk for bleeding at the procedure site. The nurse should instruct the client to remain as still as possible during the procedure to reduce the risk for puncturing the pleura or lung. The nurse should position the client in a sitting position leaning over the bedside table or laterally on the unaffected side to promote access to the site and encourage drainage of pleural fluid.

A nurse is caring for a client who is in respiratory distress. Which of the following low-flow delivery devices should the nurse use to provide the client with the highest level of oxygen? A. Nasal cannula B. Nonrebreather mask C. Simple face mask D. Partial rebreather mask

B. Nonrebreather mask Rationale: The nurse should use a nonrebreather mask for a client who is in respiratory distress to provide the highest oxygen level. A nonrebreather mask is made up of a reservoir bag from which the client obtains the oxygen, a one-way valve to prevent exhaled air from entering the reservoir bag, and exhalation ports with flaps that prevent room air from entering the mask. This device delivers greater than 90% FiO2.

A nurse in a clinic receives a phone call from a client seeking information about a new prescription for erythropoietin. Which of the following information should the nurse review with the client? A. The client needs an erythrocyte sedimentation rate (ESR) test weekly B. The client should have their hemoglobin checked twice a week C. Oxygen saturation levels should be monitored D. Folic acid production will increase

B. The client should have their hemoglobin checked twice a week

A patient with a diagnosis of pneumonia complains of a new onset of slight shortness of breath. For which of the following assessment findings would the nurse call the primary care provider immediately? (Select all that apply.) A. The patient is voiding, but the amounts are decreasing. B. The patient is sleeping more than usual. C. There is a pink coloration to the skin. D. The patient's secretions are thin and milky colored. E. The patient thought it was the 3rd instead of the 5th of the month.

B. The patient is sleeping more than usual. C. There is a pink coloration to the skin.

A nurse is performing a respiratory assessment on a patient being treated with an asthma attack. Which assessment warrants immediate action by the nurse? A. Loud Wheezing B. Increased productive cough C. Wheezing on Expiration D. Diminished breath sounds

D. Diminished breath sounds Rationale: Diminished breath sounds in a patient with asthma are an ominous sign, and the patient will typically need to be intubated when this occurs.

Which assessment is essential for the nurse to ask the patient prior to a MRI procedure? A. Do you take any sedative medications? B. Do you have a history of headaches? C. Do you have any hearing issues? D. Do you have any metal or implants in your body?

D. Do you have any metal or implants in your body?

The nurse is conducting a patient assessment. The patient tells the nurse that he has smoked two packs of cigarettes per day for 27 years. Which assessment can the nurse expect to find in this patient? A. Bounding pulse B. Night blindness C. Pupil dilate D. Elevated blood pressure

D. Elevated blood pressure

A nurse is caring for a client who has pericarditis. Which of the following findings should the nurse expect? A. Petechiae B. Murmur C. Rash D. Friction rub

D. Friction rub

A nurse is assessing a client who has chronic peripheral arterial disease (PAD). Which of the following findings should the nurse expect? A. Edema around the ankles and feet B. Ulceration around the medial malleoli C. Scaling eczema of the lower legs with stasis dermatitis D. Pallor on elevation of the limbs, and rubor when the limbs are dependent

D. Pallor on elevation of the limbs, and rubor when the limbs are dependent

A nurse in a clinic is assessing a client who has sinusitis. Which of the following techniques should the nurse use to identify manifestations of this disorder? A. Percussion of posterior lobes of lungs B. Auscultation of the trachea C. Inspection of the conjunctiva D. Palpation of the orbital areas

D. Palpation of the orbital areas

A nurse in an emergency department is assessing a client who has a bradydysrhythmia. Which of the following findings should the nurse monitor for? A. Confusion B. Friction rub C. Hypertension D. Dry skin

A. Confusion Rationale: Bradydysrhythmia can cause decreased systemic perfusion, which can lead to confusion. Therefore, the nurse should monitor the client's mental status.

What is the most likely procedure to determine the cause of severe chest pain in the patient newly admitted to the hospital? A. Coronary angiography B. Nuclear stress testing C. Right heart catheterization D. TEE

A. Coronary angiography

A patient teaching plan should focus on which risk factors for atherosclerosis? (Select all that apply.) A. Obesity B. Hyperlipidemia C. Smoking D. Age E. Race

A. Obesity B. Hyperlipidemia C. Smoking

After a percutaneous coronary angioplasty, what assessment should most concern the nurse? A. Back discomfort B. Chest pain C. Capillary refill of less than 3 seconds D. Hypoactive bowel sounds

B. Chest pain

A nurse on a cardiac unit is caring for a client who is on telemetry. The nurse recognizes the client's HR is 46 bpm and notices the provider. Which of the following prescriptions might be appropriate for this client? A. Defibrillation B. Pacemaker insertion C. Synchronized cardioversion D. Administration of IV lidocaine

B. Pacemaker insertion

A nursing is caring for a client who was admitted for treatment of left-sided heart failure and is receiving IV loop diuretics and digitalis therapy. The client is experiencing weakness and an irregular heart rate. Which of the following actions should the nurse take first? A. Obtain the client's current weight. B. Review serum electrolyte values. C. Determine the time of the last digoxin dose. D. Check the client's urine output.

B. Review serum electrolyte values. Rationale: Weakness and irregular heart rate indicate that the client is at the greatest risk for electrolyte imbalance, an adverse effect of loop diuretics. The first action the nurse should take is to review the client's electrolyte values, particularly the potassium level, because the client is at risk for dysrhythmias from hypokalemia.

A nurse is assessing a client who has left-sided heart failure. Which of the following manifestations should the nurse expect to find? A. Increased abdominal girth B. Weak peripheral pulses C. Jugular venous neck distention D. Dependent edema

B. Weak peripheral pulses Rationale: Weak peripheral pulses are related to decreased cardiac output resulting from left-sided heart failure. The other answers are all manifestations associated with right-sided heart failure.

A nurse is providing discharge teaching to a client who has COPD and a new prescription for albuterol. Which of the following statements by the client indicates understanding of the teaching? A. "This medication can increase my blood sugar levels." B. "This medication can decrease my immune response." C. "I can have an increase in my heart rate while taking this medication." D. "I can have mouth sores while taking this medication."

C. "I can have an increase in my heart rate while taking this medication."

A nurse is teaching a client who has a new prescription for ferrous sulfate. Which of the following information should the nurse include in the teaching? A. Stools will be dark red B. Take with a glass of milk if GI distress occurs C. Foods high in vitamin C will promote absorption D. Take for 14 days

C. Foods high in vitamin C will promote absorption

Carlos just finished arguing with Eleanor (mother-in-law) regarding taking over the TV and smoking in the house. Carlos is trying to quit smoking again. Shortly after he approaches Debbie and states that his head and chest hurts. He reported taking one Nitro tab 5 minutes ago. Which actions should Debbie take first? A. Take Carlos's blood pressure, tell him to avoid Eleanor and sit down B. Inquire if Carlos administered his nitroglycerin patch and take a 2nd sublingual tab C. Give him an Aspirin and another sublingual Nitro tab and call 911 D. Discuss the possibility of Eleanor living somewhere else and tell Carlos to ignore her.

C. Give him an Aspirin and another sublingual Nitro tab and call 911

Which statement by the nursing student warrants intervention by the instructor? Central Perfusion.... A. begins when the heart is stimulated by an electrical impulse B. involves the body's ability to maintain cardiac output C. involves blood flowing through arteries and capillaries to the target organs D. is impacted by preload and afterload

C. involves blood flowing through arteries and capillaries to the target organs

Current vital signs: HR 106, RR 25/min, SpO2 88%, and BP 162/98 Based upon Eleanor's manifestations, which finding supports a diagnosis of Pneumonia? A. Swelling the feet B. Not sleeping well C. Increased chest diameter D. HR 106

D. HR 106 Rationale: Swelling is more likely due to impaired perfusion. HR tells us that she is not oxygenating. With pneumonia, the biggest problem is with mucus and fluid filling the alveolar sacks and blocking oxygenation.

A nurse educator is reviewing expected findings in a client who has right-sided valvular heart disease with a group of nurses. Which of the following findings should the nurse include in the discussion? (Select all that apply.) A. Dyspnea B. Client report of fatigue C. Bradycardia D. Pleural friction rub E. Peripheral edema

A. Dyspnea B. Client report of fatigue E. Peripheral edema

A nurse is planning care for a client who has a PICC line in the right arm. Which of the following interventions should the nurse include? (Select all that apply.) A. Use a 10 mL syringe to flush the PICC line B. Apply gentle force if resistance is met during injection C. Cleanse ports with alcohol for 15 seconds prior to use D. Maintain a transparent dressing over the insertion site E. Flush with 10 mL heparin before and after medication administration

A. Use a 10 mL syringe to flush the PICC line C. Cleanse ports with alcohol for 15 seconds prior to use D. Maintain a transparent dressing over the insertion site

The nurse monitors for which complications in patients with cardiomyopathy? (Select all that apply.) A. Ventricular dysrhythmias B. Stroke C. Pericarditis D. Pulmonary embolism E. Pleural effusion

A. Ventricular dysrhythmias B. Stroke D. Pulmonary embolism

The nurse understands that it is essential for the patient to have which blood test before initiating folic acid supplementation? A. Vitamin B12 level B. Pregnancy test C. CBC D. Liver enzymes

A. Vitamin B12 level

A nurse at a provider's office is reviewing information with a client scheduled for pulmonary function tests (PFTs). Which of the following information should the nurse include? A. "Do not use inhaler medications for 6 hr following the test." B. "Do not smoke tobacco for 6-8 hr prior to the test." C. "You will be asked to bear down and hold your breath during the test." D. "The arterial blood flow to your hand will be evaluated as part of the test."

B. "Do not smoke tobacco for 6-8 hr prior to the test."

A nurse is caring for a client who is about to receive a unit of packed RBCs and states, "This is my third unit of blood today. I don't want to get some disease from all this blood." Which of the following responses should the nurse make? A. "It is impossible for you to get an infection from donor blood." B. "Donated blood is carefully screened for infectious diseases." C. "The U.S. blood supply is among the safest in the world." D. "Why not ask your doctor about other treatment options?"

B. "Donated blood is carefully screened for infectious diseases." Rationale: This statement is accurate. The nurse might continue to explain that the approach to blood safety in the U.S. includes stringent donor selection practices and the use of screening tests for HIV, AIDS, hepatitis B and C, syphilis, and other infectious diseases. Infected blood and blood products are safely discarded and are not used for transfusions.

The nurse is asked to obtain a differential after the results of the CBC reveal an elevated WBC count. Which statement is true about a differential? A. Determines the percentage of platelets in the circulating blood B. Determines the proportion of each type of WBC in a blood sample C. If the infection is bacterial, then the neutrophil count will be decreased. D. If the basophils are elevated, then the patient has a viral infection.

B. Determines the proportion of each type of WBC in a blood sample

A nurse is providing discharge teaching to a client who has a prescription for transdermal nitroglycerin patches. Which of the following instructions should the nurse include in the teaching? A. Apply the new patch to the same site as the previous patch. B. Place the patch on an area of skin away from skin folds and joints. C. Keep the patch on 24 hr per day. D. Replace the patch at the onset of angina.

B. Place the patch on an area of skin away from skin folds and joints. Rationale: The nurse should instruct the client to apply the patch to an area of intact skin with enough room for the patch to fit smoothly.

A nurse in the ED is caring for a client who is experiencing an acute asthma attack. Which of the following assessments indicates that the respiratory status is declining? (Select all that apply.) A. SaO2 95% B. Wheezing C. Retraction of sternal muscles D. Pink mucous membranes E. Tachycardia

B. Wheezing C. Retraction of sternal muscles E. Tachycardia

Which assessment is essential for the nurse to check while Eleanor is receiving Albuterol? A. temperature B. apical pulse C. weight D. intake and output

B. apical pulse Rationale: Albuterol is a beta-2 agonist, which is a sympathomimetic drug. This can cause the heart rate to become elevated.

A platelet transfusion is indicated for a patient who... A. has a systemic infection. B. has thrombocytopenia. C. is in hypovolemic shock. D. has hemolytic anemia.

B. has thrombocytopenia. Rationale: A client who has thrombocytopenia has a low platelet count. When platelet counts drop below 20,000/mm3, a transfusion of platelets is generally indicated for the client.

In providing care for a patient with potential laryngeal trauma, a priority of care is... A. monitoring oxygen saturation every 2 hours. B. having an emergency tracheostomy tray at the bedside. C. continuous cardiac monitoring. D. ensuring adequate oral intake of food.

B. having an emergency tracheostomy tray at the bedside.

A nurse is providing discharge teaching to a client who has pulmonary tuberculosis and a new prescription for rifampin. Which of the following instructions should the nurse include? A. "Ringing in the ears is an adverse effect of this medication." B. "Have your skin test repeated in 4 months to show a positive result." C. "Expect your urine and other secretions to be orange while taking this medication." D. "Remember to take this medication with a sip of water just before your first bite of each meal."

C. "Expect your urine and other secretions to be orange while taking this medication." Rationale: The nurse should inform the client that rifampin should be taken 1 hr before or 2 hr after a meal, and that it will turn urine and other secretions orange. Rifampin is hepatotoxic, so the nurse should also instruct the client to notify the provider if manifestations of hepatitis occur, including jaundice, fatigue, or malaise. The nurse should inform the client that the purified protein derivative skin test results will continue to show positive, even after the disease is no longer active.

A nurse is caring for a client who his scheduled for a coronary artery bypass graft (CABG) in 2 hr. Which of the following client statements indicates a need for further clarification by the nurse? A. "My arthritis is really bothering me because I haven't taken my aspirin in a week." B. "My blood pressure shouldn't be high because I took my blood pressure medication this morning." C. "I took my warfarin last night according to my usual schedule." D. "I will check my blood sugar because I took a reduced dose of insulin this morning."

C. "I took my warfarin last night according to my usual schedule." Rationale: Clients who are scheduled for a CABG should not take anticoagulants, such as warfarin, for several days prior to the surgery to prevent excessive bleeding.

A nurse is providing discharge teaching to a client who has a new prescription for prednisone for asthma. Which of the following client statements indicates understanding? A. "I will decrease my fluid intake while taking this medication." B. "I will expect to have black, tarry stools." C. "I will take my medication with meals." D. "I will monitor for weight loss while on this medication."

C. "I will take my medication with meals."

Which patient is at greatest risk for developing varicose veins? A. 37-year-old mail carrier B. 19-year-old retail store cashier C. 40-year-old operating room scrub nurse D. 25-year-old pregnant woman in the first trimester

C. 40-year-old operating room scrub nurse Rationale: The nurse is at risk for varicose veins because they will spend extended periods of time on their feet, and they are at a more advanced age than the cashier.

Which instruction should the nurse provide to a patient who is scheduled to have an exercise electrocardiography test? A. Have nothing to drink or eat after midnight B. Avoid smoking or drinking for at least 2 weeks before the test C. Wear comfortable, loose clothing and supportive rubber soled shoes D. Someone must drive you home because of the medication side effects.

C. Wear comfortable, loose clothing and supportive rubber soled shoes

Based upon Eleanor's condition, an ABG is obtained with the following results.. 7.28/55/80/28/89%. Which acid base imbalance is she experiencing? A. metabolic acidosis B. metabolic alkalosis C. respiratory acidosis D. respiratory alkalosis

C. respiratory acidosis Rationale: Low pH tells us acidosis, high CO2 tells us that it is a respiratory issue.

A nurse is orienting a newly licensed nurse on the care of a client who is to have a line placed for hemodynamic monitoring. Which of the following statements by the newly licensed nurse indicates understanding? A. "Air should be instilled into the monitoring system prior to the procedure." B. "The client should be positioned on the left side during the procedure." C. "The transducer should be level with the second intercostal space after the line is placed." D. "A chest x-ray is needed to verify placement after the procedure."

D. "A chest x-ray is needed to verify placement after the procedure."

A nurse is caring for a client who has a history of angina and is scheduled for exercise electrocardiography at 1100. Which of the following statements by the client requires the nurse to contact the provider for possible rescheduling? A. "I'm still hungry after the bowl of cereal I ate at 7 a.m." B. "I didn't take my heart pills this morning because the doctor told me not to." C. "I have had chest pain a couple of times since I saw my doctor in the office last week." D. "I smoked a cigarette this morning to calm my nerves about having this procedure."

D. "I smoked a cigarette this morning to calm my nerves about having this procedure." Rationale: Smoking prior to this test can change the outcome and places the client at additional risk. The procedure should be rescheduled if the client has smoked before the test.

A nurse is teaching a client who has angina about a new prescription of metoprolol. Which of the following statements by the client indicates understanding of the teaching? A. "I should place the tablet under my tongue." B. "I should have my clotting time checked weekly." C. "I will report any ringing in my ears." D. "I will call my doctor if my pulse rate is less than 60."

D. "I will call my doctor if my pulse rate is less than 60."

A nurse is providing teaching to a client who has chronic asthma and a new prescription for montelukast. Which of the following client statements indicates an understanding of the teaching? A. "I will monitor my heart rate every day while taking this medication." B. "I will make sure I have this medication with me at all times." C. "I will need to carefully rinse my mouth after I take this medication." D. "I will take this medication every night even if I don't have symptoms."

D. "I will take this medication every night even if I don't have symptoms." Rationale: Montelukast is used for the prophylactic treatment of asthma and is taken on a daily basis in the evening. Clients who take inhaled glucocorticoids should rinse their mouths and gargle after use because oral candidiasis is an adverse effect of these medications. Montelukast is not an inhaled glucocorticoid.

A nurse is reviewing discharge instructions for a client who has COPD and experienced a pneumothorax. Which of the following statements should the nurse include? A. "Notify your provider if you experience weakness." B. "You should be able to return to work in 1 week." C. "You need to wear a mask when in crowded areas." D. "Notify your provider if you develop a productive cough."

D. "Notify your provider if you develop a productive cough."

Which nursing action is indicated for the patient with thrombocytopenia? A. Avoid intramuscular injections. B. Encourage the patient to drink plenty of fluids. C. Place the patient on isolation precautions. D. Encourage frequent rest periods.

A. Avoid intramuscular injections.

Which assessment is essential for the nurse to make following a bronchoscopy? A. Check level of consciousness B. Check gag reflex C. Check neuromuscular function D. Check pedal pulses

B. Check gag reflex

The nurse understands which of the following are symptoms of hypertrophic cardiomyopathy (HCM)? (Select all that apply.) A. Weakened contraction B. Poor filling C. Decreased cardiac output D. Impaired systolic function E. Impaired diastolic function

B. Poor filling C. Decreased cardiac output E. Impaired diastolic function

The nurse is assigned to a patient with thrombocytopenia. What is the priority goal of nursing care? A. Prevention of infection B. Prevention of injury C. Prevention of dehydration D. Prevention of nutritional deficit

B. Prevention of injury

A nurse in a provider's office is assessing a client who has COPD. Which of the following findings is the priority for the nurse to report to the provider? A. Increased anterior-posterior chest diameter B. Productive cough with green sputum C. Clubbing of the fingers D. Pursed-lip breathing with exertion

B. Productive cough with green sputum Rationale: When using the urgent vs. nonurgent approach to client care, the nurse should determine that the priority finding is a productive cough with green sputum. The nurse should report this finding to the provider because it can indicate infection. The other answer choices reflect expected findings of a patient with COPD.

The nurse is aware that there are several causes of laryngitis. Which of the following can cause laryngitis? (Select all that apply.) A. Food allergies B. Upper respiratory infection C. Environmental pollutants D. Gastroesophageal reflux E. Obstructive sleep apnea

B. Upper respiratory infection C. Environmental pollutants D. Gastroesophageal reflux

T/F: The pulmonary artery contains oxygenated blood.

False

The nurse anticipates which of the following in the initial care of a patient with ARDS? A. Inotropic agents B. Fresh frozen plasma (FFP) C. Anticoagulants D. Positive-pressure ventilation

D. Positive-pressure ventilation

Which of the following is not a function of the respiratory system? A. Acid-base balance B. Temperature regulation C. Fluid balance D. Tissue perfusion

D. Tissue perfusion

A nurse is preparing to administer an initial dose of prednisone to a client with COPD. The nurse should monitor for which of the following adverse effects of this medication? (Select all that apply.) A. Hypokalemia B. Tachycardia C. Fluid retention D. Nausea E. Black, tarry stools

A. Hypokalemia C. Fluid retention E. Black, tarry stools

A nurse is assessing a client who has dilated cardiomyopathy. Which of the following findings should the nurse expect? A. Dyspnea on exertion B. Tracheal deviation C. Pericardial rub D. Weight loss

A. Dyspnea on exertion Rationale: The nurse should identify dyspnea on exertion as an expected manifestation of dilated cardiomyopathy. Dyspnea on exertion is due to ventricular compromise and reduced cardiac output.

A nurse is providing discharge teaching to a client who has heart failure. The nurse should instruct the client to report which of the following findings immediately to the provider? A. Weight gain of 0.9 kg (2 lb) in 24 hr B. Increase of 10 mm Hg in systolic blood pressure C. Dyspnea with exertion D. Dizziness when rising quickly

A. Weight gain of 0.9 kg (2 lb) in 24 hr Rationale: When using the urgent vs. nonurgent approach to client care, the nurse should determine that the priority finding is a weight gain of 0.5 to 0.9 kg (1.1 to 2 lb) in 1 day. This weight gain is an indication of fluid retention resulting from worsening heart failure. The client should report this finding immediately.

What action of intra-aortic balloon pump (IABP) therapy supports cardiac function? A. Inflating the balloon during systole, increasing CO B. Inflating the balloon during diastole, improving coronary circulation C. Deflating the balloon during diastole, decreasing SVR D. Deflating the balloon during systole, improving coronary circulation

B. Inflating the balloon during diastole, improving coronary circulation

A nurse is caring for a client who is experiencing respiratory distress. Which of the following early manifestations of hypoxemia should the nurse recognize? (Select all that apply.) A. Confusion B. Pale skin C. Bradycardia D. Hypotension E. Elevated blood pressure

B. Pale skin E. Elevated blood pressure

A nurse is completing an integumentary assessment of a client who has anemia. Which of the following findings should the nurse expect? A. Absent turgor B. Spoon-shaped nails C. Shiny, hairless legs D. Yellow mucous membranes

B. Spoon-shaped nails

A nurse is caring for a client who has a deep-vein thrombosis (DVT) and has been taking unfractionated heparin for 1 week. Two days ago, the provider also prescribed warfarin. The client asks the nurse about receiving heparin and warfarin at the same time. Which of the following statements should the nurse give? A. "I will remind your provider that you are already receiving heparin." B. "Your laboratory findings indicated that two anticoagulants were needed." C. "It takes 3 to 4 days before the therapeutic effects of the warfarin are achieved, and then the heparin can be discontinued." D. "Only one of these medications is being given to treat your DVT."

C. "It takes 3 to 4 days before the therapeutic effects of the warfarin are achieved, and then the heparin can be discontinued."

What is an important nursing action following a cardiac catheterization intervention? A. Early mobilization to prevent clot formation B. Fluid restriction to avoid fluid overload C. Bedrest to avoid stress on cannula insertion site D. Head of bed at 30 degrees for respiratory support

C. Bedrest to avoid stress on cannula insertion site

What action will most help a client obtain maximum benefits after postural drainage? A. Administer PRN Oxygen B. Place the client in a sitting position C. Encourage the client to cough deeply D. Encourage the client to rest for 30 minutes

C. Encourage the client to cough deeply Rationale: Gravity is the main contributor to postural drainage. We want the patient to be able to cough up the mucus.

A nurse is providing education to a client who has a prescription for a blood transfusion. Which of the following statements should the nurse include in the teaching? A. "I will check your vital signs every 15 minutes throughout the blood transfusion." B. "I might have a nursing assistant check on you periodically during the transfusion." C. "If you have no adverse effects in the first 15 to 30 minutes, you will not have any adverse effects later." D. "You must immediately report any symptoms like chills, nausea, or itching."

D. "You must immediately report any symptoms like chills, nausea, or itching." Rationale: Although the nurse can identify objective signs of a transfusion reaction (changes in vital signs, flushing, cyanosis, coughing, and to some extent, dyspnea), the nurse might not be able to tell if the client is experiencing subjective symptoms (chills, nausea, chest pain, headache, backache, muscle pain). Subjective signs are important clues, and the nurse must be aware of them.

A nurse is teaching a client who is scheduled for coronary angiography. Which of the following statements should the nurse include? A. "You should have nothing to eat or drink for 4 hr prior to the procedure." B. "You will be given general anesthesia during the procedure." C. "You should not have this procedure done if you are allergic to eggs." D. "You will need to keep your affected leg straight following the procedure."

D. "You will need to keep your affected leg straight following the procedure."

The nurse is caring for a patient on a heparin drip who was admitted for DVT 2 days ago. Which laboratory value is most important to report to the provider immediately? A. A normal INR B. An increased hematocrit C. An increased platelet count D. A normal aPTT

D. A normal aPTT

A nurse is caring for a client following insertion of a permanent pacemaker. Which of the following client statements indicates a potential complication of the insertion procedure? A. "I can't get rid of these hiccups." B. "I feel dizzy when I stand." C. "My incision site stings." D. "I have a headache."

A. "I can't get rid of these hiccups." Rationale: Hiccups can indicate that the pacemaker is stimulating the chest wall or diaphragm, which can occur as a result of a lead wire perforation.

A nurse is providing patient education regarding OSA to a newly diagnosed patient. Patient understanding is indicated by which of the following statements? A. "I guess I should really lose weight." B. "I do not have to lose weight as long as I can use CPAP at night." C. "A glass of wine in the evening will help me sleep through the night." D. "I will not have to take my blood pressure meds if I use CPAP at night."

A. "I guess I should really lose weight."

Which statements by a patient with a AAA indicate that teaching has been effective? (Select all that apply.) A. "I need to quit smoking." B. "I need to go to the emergency department immediately if I have new severe abdominal pain." C. "The doctor may put me on blood thinners." D. "I need to stay on my blood pressure medication." E. "I should keep my legs elevated whenever possible."

A. "I need to quit smoking." B. "I need to go to the emergency department immediately if I have new severe abdominal pain." D. "I need to stay on my blood pressure medication."

Eleanor is receiving Albuterol and Flovent via inhalers. Which patient statement requires further teaching by the nurse? (Select all that apply.) A. "I will wait at least 5 minutes between each puff of my Flovent" B. "I will rinse my mouth after the Albuterol" C. "I should take the Flovent at the first sign of respiratory troubles" D. "I will call the doctor if I feel any chest discomfort with the medications"

A. "I will wait at least 5 minutes between each puff of my Flovent" B. "I will rinse my mouth after the Albuterol" C. "I should take the Flovent at the first sign of respiratory troubles" Rationale: Patients only have to wait 1-2 minutes between Flovent puffs. The patient should rinse their mouth out after Flovent and other inhaled glucocorticoids. Flovent is not a rescue medication, but rather a controller medication that should be taken daily to prevent exacerbations.

A nurse is providing discharge teaching for a client who has heart failure and is on a fluid restriction of 2,000 mL/day. The client asks the nurse how to determine the appropriate amount of fluids they are allowed. Which of the following statements is an appropriate response by the nurse? A. "Pour the amount of fluid you drink into an empty 2-liter bottle to keep track of how much you drink." B. "Each glass contains 8 ounces. There are 30 milliliters per ounce, so you can have a total of 8 glasses or cups of fluid each day." C. "This is the same as 2 quarts, or about the same as two pots of coffee." D. "Take sips of water or ice chips so you will not take in too much fluid."

A. "Pour the amount of fluid you drink into an empty 2-liter bottle to keep track of how much you drink."

A nurse is discharging a client who has COPD. The client is concerned about not being able to leave the house due to the need for staying on continuous oxygen. Which of the following responses should the nurse make? A. "There are portable oxygen delivery systems you can take with you." B. "When you go out, you can remove the oxygen and then reapply it when you get home." C. "You probably will not be able to go out as much as you used to." D. "Home health services will come to you so you will not need to get out."

A. "There are portable oxygen delivery systems you can take with you."

A nurse is providing teaching to a client who is 2 days postoperative following a heart transplant. Which of the following statements should the nurse include in the teaching? A. "You might no longer be able to feel chest pain." B. "Your level of activity intolerance will not change." C. "After 6 months, you will no longer need to restrict your sodium intake." D. "You will be able to stop taking immunosuppressants after 12 months."

A. "You might no longer be able to feel chest pain." Rationale: Heart transplant clients usually are no longer able to feel chest pain due to the denervation of the heart. The client will need to permanently maintain a diet that is restricted in sodium and fat. The client will remain on immunosuppressants for the remainder of their life to help prevent rejection of the heart.

A nurse is providing discharge teaching to a client who had a gastrectomy for stomach cancer. Which of the following information should the nurse include in the teaching? (Select all that apply.) A. "You will need a monthly injection of vitamin B12 for the rest of your life." B. "Using the nasal spray form of vitamin B12 on a daily basis can be an option." C. "An oral supplement of vitamin B12 taken on a daily basis can be an option." D. "You should increase your intake of animal proteins, legumes, and dairy products to increase vitamin B12 in your diet." E. "Add soy milk fortified with vitamin B12 to your diet to decrease the risk of pernicious anemia."

A. "You will need a monthly injection of vitamin B12 for the rest of your life." B. "Using the nasal spray form of vitamin B12 on a daily basis can be an option."

The nurse is screening patients for their risk of developing hypertension. The nurse should consider which patients at greatest risk? (Select all that apply.) A. A 40-year-old Latino male who is obese and smokes two packs of cigarettes per day B. A 35-year-old Asian female who has a familial history of diabetes mellitus type 1 C. A 78-year-old African American male with chronic renal insufficiency D. A 25-year-old African American female track athlete with a healthy body mass index (BMI) who takes oral contraceptives E. A 60-year-old Caucasian male with vitamin D deficiency and a history of cocaine use

A. A 40-year-old Latino male who is obese and smokes two packs of cigarettes per day C. A 78-year-old African American male with chronic renal insufficiency E. A 60-year-old Caucasian male with vitamin D deficiency and a history of cocaine use

A nurse is reviewing the health records of five clients. Which of the following clients are at risk for developing acute respiratory distress syndrome? (Select all that apply.) A. A client who experienced a near-drowning incident B. A client following coronary artery bypass graft surgery C. A client who has a hemoglobin of 15.1 mg/dL D. A client who has dysphagia E. A client who experienced acute drug toxicity

A. A client who experienced a near-drowning incident B. A client following coronary artery bypass graft surgery D. A client who has dysphagia E. A client who experienced acute drug toxicity

A nurse receives prescriptions from the provider for performing nasopharyngeal suctioning on four clients. For which of the following clients should the nurse clarify the provider's prescription? A. A client who has epistaxis B. A client who has amyotrophic lateral sclerosis C. A client who has pneumonia D. A client who has emphysema

A. A client who has epistaxis Rationale: The nurse should avoid providing nasopharyngeal suctioning for a client who has nasal bleeding because this intervention might cause an increase in bleeding.

A nurse is on a cardiac unit caring for a group of clients. The nurse should recognize which of the following clients as being at risk for the development of a dysrhythmia? (Select all that apply.) A. A client who has metabolic acidosis B. A client who has a blood potassium level of 4.3 mEq/L C. A client who has an SaO2 of 96% D. A client who has COPD E. A client who underwent stent placement in a coronary artery

A. A client who has metabolic acidosis D. A client who has COPD E. A client who underwent stent placement in a coronary artery

A nurse is caring for four clients. Which of the following clients is at greatest risk for pulmonary embolism? A. A client who is 48 hr postoperative following a total hip arthroplasty B. A client who is 8 hr postoperative following an open surgical appendectomy C. A client who is 2 hr postoperative following an open reduction external fixation of the right radius D. A client who is 4 hr postoperative following a laparoscopic cholecystectomy

A. A client who is 48 hr postoperative following a total hip arthroplasty Rationale: The nurse should identify that a client who has undergone a total hip arthroplasty surgery is at greatest risk for a pulmonary embolus because of decreased mobility of the affected extremity and an increased amount of blood clots forming in the veins of the thigh following hip surgery. Deep-vein thromboses are most likely to occur 48 to 72 hr following the arthroplasty. The nurse should intervene to reduce the risk by applying sequential compression devices or antiembolic stockings and by administering anticoagulant medications.

A nurse is caring for a patient with a diagnosis of MI. The patient calls the nurse because he is experiencing chest pain. The nurse administers an SL nitroglycerin tablet as prescribed. After 5 minutes, the chest pain is unrelieved by the nitroglycerin. The next nursing action is which of the following? A. Administer another nitroglycerin tablet B. Increase the flow rate of the oxygen C. Contact the provider D. Call the charge nurse

A. Administer another nitroglycerin tablet

A nurse is developing a plan of care for a client who has active tuberculosis. Which of the following isolation precautions should the nurse include in the plan? A. Airborne B. Neutropenic C. Contact D. Droplet

A. Airborne Rationale: The nurse should initiate airborne precautions for a client who has tuberculosis because tuberculosis is a respiratory infection that is spread through the air. The client should be placed in a room with negative airflow pressure that is filtered through a high-efficiency particulate air (HEPA) filter. Members of the health care team should not enter the client's room without wearing an N95 respirator mask.

The nurse is taking a history and vital signs on the patient with fatigue. The nurse notes a regular apical pulse of 110 beat/min. Which contributing factor does the nurse assess as a possible rationale for the patient's condition? (Select all that apply.) A. Anxiety B. Stress C. Hypovolemia D. Anemia E. Hypothyroidism

A. Anxiety B. Stress C. Hypovolemia D. Anemia Rationale: Hypothyroidism will cause bradycardia, not tachycardia.

A 68-year-old male presents to the emergency department with complaints of crushing chest pain that radiates to the left shoulder. The patient is diagnosed with AMI. Admission orders include oxygen 2 L via nasal cannula, blood work, chest x-ray, 12-lead ECG, and SL nitroglycerin. What should be the nurse's first action? A. Apply oxygen B. Obtain the 12-lead ECG C. Administer the nitroglycerin D. Obtain the blood work

A. Apply oxygen

A nurse in an ED is caring for a client who is experiencing a pulmonary embolism. Which of the following actions should the nurse take first? A. Apply supplemental oxygen. B. Increase the rate of IV fluids. C. Administer pain medication. D. Initiate cardiac monitoring.

A. Apply supplemental oxygen. Rationale: When using the airway, breathing, circulation approach to client care, the greatest risk to the client is severe hypoxemia. The nurse should increase the rate of the IV fluids to increase cardiac output, administer pain medications to decrease discomfort and anxiety, and initiate cardiac monitoring because the client is at risk for dysrhythmias and right ventricular failure. However, applying supplemental oxygen takes priority.

After a bronchoscopy, the patient coughs up sputum which contains blood. What is the best nursing intervention for this patient? A. Assess vital signs and respiratory status and notify physician B. Monitor the patient to see if the blood continues in the sputum C. Send the sputum to the lab for cytology D. Reassure the patient that bleeding can be expected after a bronchoscopy

A. Assess vital signs and respiratory status and notify physician Rationale: Only blood TINGED sputum is expected after a bronchoscopy. The physician should be notified and vital signs should be taken

A nurse is assessing a client who has lung cancer. Which of the following manifestations should the nurse expect? A. Blood-tinged sputum B. Decreased tactile fremitus C. Resonance with percussion D. Peripheral edema

A. Blood-tinged sputum Rationale: The nurse should expect blood-tinged sputum secondary to bleeding from the tumor. Additionally, the nurse should expect an increase in tactile fremitus, dullness with percussion, and cyanosis of the lips and fingertips.

The nurse on the 3-11 shift received report and is caring for three patents in addition to Carlos. Which patient should the nurse see first? A. Carlos scheduled for discharge tomorrow, reported vomiting a few minutes ago and is complaining of muscle weakness B. Mike newly diagnosed with hypertension, is awaiting medication teaching and needs an echocardiogram C. Rocco 3 days post MI is constipated, anxious and demanding that the doctor be notified D. Ann admitted with acute coronary syndrome received her first dose of propranolol (Inderal) and aspirin this morning

A. Carlos scheduled for discharge tomorrow, reported vomiting a few minutes ago and is complaining of muscle weakness Rationale: While the other patients should also be seen, they are all stable. Carlos has had a change in his condition and should be assessed for a possible explanation for these newly onset symptoms.

Carlos 's stress test reflects angina, and he is ordered Nitroglycerin SL PRN for pain and Nitroglycerin transdermal patch to apply in AM and remove at bedtime. Which instruction should the nurse provide regarding these medications? A. Change position slowly B. Chew the nitroglycerin tab to enhance absorption C. Store the Nitroglycerin patches in the refrigerator D. You cannot take both the patch and tablets at the same time.

A. Change position slowly Rationale: Nitroglycerin is associated with orthostatic hypotension. The Nitroglycerin SL should not be chewed, but rather dissolved underneath the tongue.

A nurse is monitoring a group of clients for increased risk for developing pneumonia. Which of the following clients should the nurse expect to be at risk? (Select all that apply.) A. Client who has dysphagia B. Client who has AIDS C. Client who was vaccinated for pneumococcus and influenza 6 months ago D. Client who is postoperative and has received local anesthesia E. Client who has a closed head injury and is receiving mechanical ventilation F. Client who has myasthenia gravis

A. Client who has dysphagia B. Client who has AIDS E. Client who has a closed head injury and is receiving mechanical ventilation F. Client who has myasthenia gravis

A nurse is preparing to initiate the transfusion of a unit of packed RBCs to a client. After the unit of blood has arrived, which of the following procedures will help protect the client against the possibility of a blood-group incompatibility? A. Comparing the ID numbers on the blood unit with those on the order form and the client's wristband B. Obtaining a blood sample from the client for typing and crossmatching C. Giving a prescribed pre-medication 30 min prior to starting the transfusion D. Observing the client for 15 to 30 min after the transfusion is initiated

A. Comparing the ID numbers on the blood unit with those on the order form and the client's wristband Rationale: Before administering blood or blood products, the client's nurse and one other authorized individual are required to identify the client and compare the label on the blood product to the prescription in the medical record and the client's armband. These actions will verify that the correct client is receiving the correct blood and help prevent the possibility of a blood-group incompatibility.

The nurse recognizes that a deficiency in a clotting factor may cause which finding(s)? (Select all that apply.) A. Easy bruising and cutaneous hematoma formation with minor trauma (e.g., an injection) B. Bleeding from the gums and prolonged bleeding following minor injuries or cuts C. Enhanced platelet aggregation and increased clumping of RBCs D. Fibrin molecules form fibrin threads to increase wound healing E. Yellowish skin color

A. Easy bruising and cutaneous hematoma formation with minor trauma (e.g., an injection) B. Bleeding from the gums and prolonged bleeding following minor injuries or cuts

Which diagnostic test can the nurse expect the health care provider will order first for the patient with a heart murmur? A. Echocardiogram B. Holter monitor C. Cardiac catheterization D. Stress test

A. Echocardiogram Rationale: This test is noninvasive and allows visualization of the cardiac structures.

The nursing plan of care for a patient diagnosed with lung cancer includes which of the following? (Select all that apply.) A. Education about disease management B. Frequent assessment of pain levels C. Reminders that recovery is coming soon D. Frequent assessment of oxygenation E. Weaning from oxygen because the disease is terminal

A. Education about disease management B. Frequent assessment of pain levels D. Frequent assessment of oxygenation

A nurse is assessing a patient in the emergency department with the complaint of sudden onset of severe back pain, tachycardia, and hypotension. Which interventions should the nurse anticipate? (Select all that apply.) A. Electrocardiogram B. Aortic arteriography C. Ultrasonography D. Chest x-ray E. Computed tomography scan

A. Electrocardiogram C. Ultrasonography E. Computed tomography scan

A nurse is planning care for a client following the insertion of a chest tube and drainage system. Which of the following should be included in the plan of care? (Select all that apply.) A. Encourage the client to cough and deep breathe B. Check for continuous bubbling in the suction chamber C. Strip the drainage tubing every 4 hr D. Clamp the tube once a day E. Obtain a chest x-ray

A. Encourage the client to cough and deep breathe B. Check for continuous bubbling in the suction chamber E. Obtain a chest x-ray

A nurse is caring for a client who experienced defibrillation. Which of the following should be included in the documentation of this procedure? (Select all that apply.) A. Follow-up ECG B. Energy settings used C. IV fluid intake D. Urinary output E. Skin condition under electrodes

A. Follow-up ECG B. Energy settings used E. Skin condition under electrodes

What is the composition of the heart? A. Four chambers with four valves that control flow through the heart and lungs through changes in pressure B. Four chambers and four valves that control flow through the heart and lungs through changes in oxygen levels C. Two chambers on the right receiving blood from the high-pressure venous system and two chambers on the left sending blood into the low-pressure arterial system D. Two chambers on the right receiving oxygenated blood from the venous system and two chambers on the left receiving deoxygenated blood from the pulmonary circuit

A. Four chambers with four valves that control flow through the heart and lungs through changes in pressure

A nurse is caring for a client who is to receive thrombolytic therapy. Which of the following factors should the nurse recognize as a contraindication to the therapy? A. Hip arthroplasty 2 weeks ago B. Elevated sedimentation rate C. Incident of exercise-induced asthma 1 week ago D. Elevated platelet count

A. Hip arthroplasty 2 weeks ago

The nurse understands that oxygen therapy for a patient with COPD requires close monitoring because of which of the following? A. Hypoxic respiratory drive B. Hypercapnic respiratory drive C. Acidotic respiratory drive D. Alkalotic respiratory drive

A. Hypoxic respiratory drive

A nurse is preparing a client for discharge following a bronchoscopy with the use of moderate sedation. The nurse should identify that which of the following assessments is the priority? A. Presence of gag reflex B. Pain level rating using a 0 to 10 scale C. Hydration status D. Appearance of the IV insertion site

A. Presence of gag reflex Rationale: The greatest risk to the client is aspiration due to a depressed gag reflex. Therefore, the priority assessment by the nurse is to determine the return of the gag reflex.

Current status: HR 106, RR 25/min, SpO2 88%, and BP 162/98, respiratory acidosis Based upon the ABG results and Eleanor's manifestations, which action should the nurse implement? (Select all that apply.) A. Increase the head of the bed B. Increase the oxygen to keep pulse ox >95% C. Bedrest D. Encourage fluids at least 3L /day

A. Increase the head of the bed C. Bedrest Rationale: Though she does not have an official COPD diagnosis, we want to err on the side of caution and administer low-flow O2 so as not to impair her oxygen drive. We will increase the head of the bed to allow for increased oxygenation, and encourage bedrest to conserve energy. Since her blood pressure is already elevated, we will want to restrict fluids to prevent any fluid retention.

A nurse is assessing a client who has splinter hemorrhages of the nail beds and reports a fever. The nurse should identify these findings as manifestations of which of the following disorders? A. Infective endocarditis B. Pericarditis C. Myocarditis D. Rheumatic endocarditis

A. Infective endocarditis

The patient is scheduled for a cardiac catheterization. Which interventions are appropriate by the nurse? (Select all that apply.) A. Instruction the patient that a warm sensation may be felt during the procedure B. Inform patient to report pain while the catheter is being inserted C. Instruct the patient that medications will be given to induce sleep D. Check the patient for allergies to shellfish

A. Instruction the patient that a warm sensation may be felt during the procedure B. Inform patient to report pain while the catheter is being inserted D. Check the patient for allergies to shellfish

The instructor is monitoring a student who is caring for a patient returning to the unit from a cardiac catheterization(femoral vein used). Which student action requires the nursing instructor to intervene? The student... A. Instructs the patient to remain in bed for 24 hours B. Assesses the pedal pulses and skin for temperature and color C. Monitors the intake urine output D. Checks the patient's groin pressure dressing

A. Instructs the patient to remain in bed for 24 hours

Age-related changes that affect the hematological system include which findings? (Select all that apply.) A. Iron binding decreases. B. The number of stem cells in the marrow increases. C. Lymphocyte function, especially cellular immunity, decreases. D. Platelet adhesiveness decreases. E. Hematocrit decreases.

A. Iron binding decreases. C. Lymphocyte function, especially cellular immunity, decreases.

The nurse recognizes which statement as correct about blood cell formation? A. It occurs mostly in the marrow found in flat bones such as the sternum, ribs, and pelvis B. It occurs mostly in the marrow found in the shaft of long bones C. It occurs outside the marrow, once it enters the circulatory system D. It occurs after birth until late adolescence

A. It occurs mostly in the marrow found in flat bones such as the sternum, ribs, and pelvis

A nurse is planning a presentation for a group of clients who have hypertension. Which of the following lifestyle modifications should the nurse include? (Select all that apply.) A. Limited alcohol intake B. Regular exercise program C. Decreased magnesium intake D. Reduced potassium intake E. Tobacco cessation

A. Limited alcohol intake B. Regular exercise program E. Tobacco cessation Rationale: Low magnesium intake and low potassium intake are both associated with hypertension and are not lifestyle modifications the nurse should include.

A nurse is caring for a client who, upon awakening, was disoriented to person, place and time. The client reports chills and chest pain that is worse upon inspiration. Which of the following actions is the nursing priority? A. Obtain baseline vital signs and oxygen saturation B. Obtain a sputum culture C. Obtain a complete history from the client D. Provide a pneumococcal vaccine

A. Obtain baseline vital signs and oxygen saturation

Potential complications of rhinosinusitis include which of the following? (Select all that apply.) A. Orbital cellulitis B. Osteomyelitis C. Meningitis D. Pharyngitis E. Orbital abscess

A. Orbital cellulitis B. Osteomyelitis D. Pharyngitis

A nurse is preparing to care for a client following chest tube placement. Which of the following items should be available in the client's room? (Select all that apply.) A. Oxygen B. Sterile water C. Enclosed hemostat clamps D. Indwelling urinary catheter E. Occlusive dressing

A. Oxygen B. Sterile water C. Enclosed hemostat clamps E. Occlusive dressing

A nurse is caring for a client who is scheduled for a thoracentesis. Which of the following supplies should the nurse ensure are in the client's room? (Select all that apply.) A. Oxygen equipment B. Incentive spirometer C. Pulse oximeter D. Sterile dressing E. Suture removal kit

A. Oxygen equipment C. Pulse oximeter D. Sterile dressing

Which patients may require a CT scan? (Select all that apply.) A. Patient with suspected abdominal bleeding B. Patient with suspected pulmonary embolism C. Patient with fever and a bladder infection D. A pregnant female with a fracture femur

A. Patient with suspected abdominal bleeding B. Patient with suspected pulmonary embolism

A nurse is providing information about tuberculosis to a group of clients at a local community center. Which of the following manifestations should the nurse include? (Select all that apply.) A. Persistent cough B. Weight gain C. Fatigue D. Night sweats E. Purulent sputum

A. Persistent cough C. Fatigue D. Night sweats E. Purulent sputum

A nurse in a clinic is caring for a client who has been on long-term NSAID therapy to treat pericarditis. Which of the following laboratory findings should the nurse report to the provider? A. Platelets 100,000/mm3 B. Serum glucose 110 mg/dL C. Serum creatinine 0.7 mg/dL D. Aminoalanine transferase (ALT) 30IU/L

A. Platelets 100,000/mm3

A nurse is caring for a client who is scheduled for a thoracentesis. Prior to the procedure, which of the following actions should the nurse take? A. Position the client in an upright position, leaning over the bedside table. B. Explain the procedure. C. Obtain ABGs. D. Administer benzocaine spray

A. Position the client in an upright position, leaning over the bedside table.

A nurse is planning care for a client who has Hgb 7.5 g/dL and Hct 21.5%. Which of the following actions should the nurse include in the plan of care? (Select all that apply.) A. Provide assistance with ambulating B. Monitor oxygen saturation C. Weigh the client weekly D. Obtain stool specimen for occult blood E. Schedule daily rest periods

A. Provide assistance with ambulating B. Monitor oxygen saturation D. Obtain stool specimen for occult blood E. Schedule daily rest periods

The new RN is administering a blood product under the observation of the charge nurse. Which actions by the new nurse warrant intervention by the charge nurse? (Select all that apply.) A. Provides IV D5NSS solution primed along with the blood product B. Stays with the patient during the first 15 minutes of the transfusion C. Once the blood is connected to the patient the nurse sets the pump to the ordered infusion rate D. Takes vital signs at the start and 15 minutes into the infusion

A. Provides IV D5NSS solution primed along with the blood product C. Once the blood is connected to the patient the nurse sets the pump to the ordered infusion rate

A nurse is caring for a client who has heart failure and is experiencing atrial fibrillation. Which of the following findings should the nurse plan to monitor for and report to the provider immediately? A. Slurred speech B. Irregular pulse C. Dependent edema D. Persistent fatigue

A. Slurred speech Rationale: The greatest risk to this client is injury from an embolus caused by the pooling of blood that can occur with atrial fibrillation. Slurred speech can indicate inadequate circulation to the brain because of an embolus. Therefore, the nurse should report this finding to the provider immediately.

A nurse is admitting a client who has a suspected myocardial infarction (MI) and a history of angina. Which of the following findings will help the nurse distinguish stable angina from an MI? A. Stable angina can be relieved by rest and nitroglycerin B. The pain of an MI resolves in less than 15 min C. The type of activity that causes an MI can be identified D. Stable angina can occur for longer than 30 min

A. Stable angina can be relieved by rest and nitroglycerin

A nurse is caring for a client who is receiving a blood transfusion. Which of the following actions should the nurse plan to take if an allergic transfusion reaction is suspected? (Select all that apply.) A. Stop the transfusion B. Monitor for hypertension C. Maintain an IV infusion with 0.9% sodium chloride D. Position the client in an upright position with the feet lower than the heart E. Administer diphenhydramine

A. Stop the transfusion C. Maintain an IV infusion with 0.9% sodium chloride E. Administer diphenhydramine

A nurse is providing care for a patient newly diagnosed with heart disease. Which dietary, activity, or lifestyle modification(s) should be included in the plan of care? (Select all that apply.) A. Stopping smoking B. Drinking lots of water C. Limiting sedentary lifestyle D. Eating a diet rich in red meat and protein E. Limiting alcohol intake

A. Stopping smoking C. Limiting sedentary lifestyle E. Limiting alcohol intake

A nurse is reviewing the health record of a client who is being evaluated for possible valvular heart disease. The nurse should recognize which of the following data as risk factors for this condition? (Select all that apply.) A. Surgical repair of an atrial septal defect at age 2 B. Measles infection during childhood C. Hypertension for 5 years D. Weight gain of 10 lb in past year E. Diastolic murmur present

A. Surgical repair of an atrial septal defect at age 2 C. Hypertension for 5 years E. Diastolic murmur present

A nurse is completing the admission assessment of a client who has suspected pulmonary edema. Which of the following manifestations are expected findings? (Select all that apply.) A. Tachypnea B. Persistent cough C. Increased urinary output D. Thick, yellow sputum E. Orthopnea

A. Tachypnea B. Persistent cough E. Orthopnea

You are caring for a patient who is currently receiving continuous pulse oximetry. Which of the following findings might indicate inaccurate readings? A. The patient has a core body temperature of 94°F (34.5 °C). B. The heart rate obtained through pulse oximetry correlates with the electrocardiogram heart rate. C. The patient has the probe attached to her earlobe. D. The patient's pulse oximetry is 95%, and the patient denies shortness of breath.

A. The patient has a core body temperature of 94°F (34.5 °C).

A nurse at a provider's office is reviewing the laboratory test results for a group of clients. The nurse should identify that which of the following results indicates the client is at risk for heart disease? (Select all that apply.) A. Total cholesterol 245 mg/dL B. HDL 90 mg/dL C. LDL 140 mg/dL D. Triglycerides 125 mg/dL E. Troponin I 0.02 ng/mL

A. Total cholesterol 245 mg/dL C. LDL 140 mg/dL

Which tests can the nurse anticipate will be ordered for the patient admitted with chest pain and dyspnea? (Select all that apply.) A. Troponin B. D-Dimer C. Electrolytes D. Albumin

A. Troponin B. D-Dimer C. Electrolytes Rationale: Troponin will be taken to assess if any myocyte damage has occurred. D-Dimer is done to assess the presence of a blood clot, such as in a pulmonary embolism. Electrolytes such as K+ and Ca+ have an impact on cardiac contractility.

The patient with type B+ blood is scheduled to receive packed red blood cells. Which blood types can the nurse transfuse safely to this patient? (Select all that apply.) A. Type B- B. Type A+ C. Type O+ D. Type AB+

A. Type B- C. Type O+

When should palliative care be initiated for patients diagnosed with lung cancer? A. Upon diagnosis B. After chemotherapy C. After surgery D. Upon patient request

A. Upon diagnosis

Carlos is placed on an ACE inhibitor for his hypertension. Which statement by Carlos requires intervention by the nurse? A. "I will remember to take this medication every day in the morning before I leave for work." B. "I am going to limit the fatty snacks and eat more fruits such as cantaloupes and bananas." C. "This medication may make me urinate more since it works with the kidney." D. "I may develop a cough with this medication and will notify the doctor if one develops".

B. "I am going to limit the fatty snacks and eat more fruits such as cantaloupes and bananas." Rationale: Patients on ACE inhibitors should avoid foods high in potassium due to the risk of hyperkalemia associated with these medications.

A nurse is caring for a client who has a new prescription for heparin therapy. Which of the following statements by the client should indicate an immediate concern for the nurse? A. "I am allergic to morphine." B. "I take antacids several times a day for my ulcer." C. "I had a blood clot in my leg several years ago." D. "It hurts to take a deep breath."

B. "I take antacids several times a day for my ulcer." Rationale: The greatest risk to the client is the possibility of bleeding from a peptic ulcer. The priority intervention is to notify the provider of the finding.

A nurse is reinforcing teaching with a client on the purpose of taking a bronchodilator. Which of the following client statements indicates understanding? A. "This medication can decrease my immune response." B. "I take this medication to prevent asthma attacks." C. "I need to take this medication with food." D. "This medication has a slow onset to treat my symptoms."

B. "I take this medication to prevent asthma attacks."

The student nurse is reviewing care of patients post bronchoscopy. Which statement by the student warrants action by the instructor? A. "I will check the pulse ox frequently." B. "I will encourage fluids to flush out the dye." C. "I will check the sputum for blood." D. "I will monitor for any stridor or wheezing."

B. "I will encourage fluids to flush out the dye."

A nurse is completing discharge teaching with a client who had a surgical placement of a mechanical heart valve. Which of the following statements by the client indicates understanding of the teaching? A. "I will be glad to get back to my exercise routine right away." B. "I will have my prothrombin time checked on a regular basis." C. "I will talk to my dentist about no longer needing antibiotics before dental exams." D. "I will continue to limit my intake of foods containing potassium."

B. "I will have my prothrombin time checked on a regular basis."

A nurse is providing teaching for a client who is scheduled for a bone marrow biopsy of the iliac crest. Which of the following statements made by the client indicates understanding of the teaching? A. "This test will be performed while I am lying flat on my back." B. "I will need to stay in bed for about an hour after the test." C. "This test will determine which antibiotic I should take for treatment." D. "I will receive general anesthetic for the test."

B. "I will need to stay in bed for about an hour after the test."

A charge nurse is reviewing the care of a client who has a chest tube connected to a water seal drainage system in place following thoracic surgery with a newly licensed nurse. Which of the following statements by the newly licensed nurse indicates an understanding of when to notify the provider? A. "I will notify the provider if there is a fluctuation of drainage in the tubing with inspiration." B. "I will notify the provider if there is continuous bubbling in the water seal chamber." C. "I will notify the provider if there is drainage of 60 milliliters in the first hour after surgery." D. "I will notify the provider if there are several small, dark-red blood clots in the tubing."

B. "I will notify the provider if there is continuous bubbling in the water seal chamber." Rationale: Continuous bubbling in the water seal chamber suggests an air leak and requires notification of the provider. The nurse should check the system for external, correctable leaks while waiting for instructions from the provider.

A nurse is providing teaching for a client who has a new diagnosis of hypertension and a new prescription for spironolactone 25 mg/day. Which of the following statements by the client indicates an understanding of the teaching? A. "I should eat a lot of fruits and vegetables, especially bananas and potatoes." B. "I will report any changes in heart rate to my provider." C. "I should replace the salt shaker on my table with a salt substitute." D. "I will decrease the dose of this medication when I no longer have headaches and facial redness."

B. "I will report any changes in heart rate to my provider."

A charge nurse is providing an in-service to a group of staff nurses about endotracheal suctioning. Which of the following statements by a staff nurse indicates an understanding of the teaching? A. "I will use clean technique when suctioning a client's endotracheal tube." B. "I will use a rotating motion when removing the suction catheter." C. "I will suction the oropharyngeal cavity prior to suctioning the endotracheal tube." D. "I will suction a client's endotracheal tube every 2 hours."

B. "I will use a rotating motion when removing the suction catheter." Rationale: The nurse should rotate the suction catheter during withdrawal to remove secretions from the sides of the airway. The nurse should use sterile technique when performing endotracheal suctioning to avoid the introduction of pathogens into the sterile respiratory system. The nurse should suction the endotracheal tube prior to suctioning the nonsterile oropharyngeal cavity to prevent cross contamination.

A home health nurse is teaching a client who has active tuberculosis and is following a medication regimen that includes a combination of isoniazid, rifampin, pyrazinamide, and ethambutol. Which of the following client statements indicates understanding? (Select all that apply.) A. "I can substitute one medication for another if I run out because they all fight infection." B. "I will wash my hands each time I cough." C. "I will wear a mask when I am in a public area." D. "I am glad I don't have to have any more sputum specimens." E. "I don't need to worry where I go once I start taking my medications."

B. "I will wash my hands each time I cough." C. "I will wear a mask when I am in a public area."

A nurse is orienting a newly licensed nurse on the purpose of administering vecuronium to a client who has acute respiratory distress syndrome (ARDS). Which of the following statements by the newly licensed nurse indicates understanding of the teaching? A. "This medication is given to treat infection." B. "The medication is given to facilitate ventilation." C. "This medication is given to decrease inflammation." D. "This medication is given to reduce anxiety."

B. "The medication is given to facilitate ventilation."

A nurse is caring for an older adult client who is to undergo a percutaneous balloon valvuloplasty. The client's family member asks the nurse to explain the expected outcome of this procedure. Which of the following responses should the nurse give? A. "This will improve blood flow to the coronary arteries." B. "This will assist with the ability to perform activities of daily living." C. "This will prolong the life span of living with this valve disorder." D. "This will reverse the effects to the damaged area."

B. "This will assist with the ability to perform activities of daily living."

A nurse is teaching a client who has tuberculosis. Which of the following statements should the nurse include? A. "You will need to continue to take the multimedication regimen for 4 months." B. "You will need to provide sputum samples every 4 weeks to monitor the effectiveness of the medication." C. "You will need to remain hospitalized for treatment." D. "You will need to wear a mask at all times."

B. "You will need to provide sputum samples every 4 weeks to monitor the effectiveness of the medication."

A nurse is providing health teaching to a group of clients. Which of the following clients is at risk for developing peripheral arterial disease? A. A client who has hypothyroidism B. A client who has diabetes mellitus C. A client whose daily caloric intake consists of 25% fat D. A client who consumes two 12-oz (0.35-L) bottles of beer a day

B. A client who has diabetes mellitus Rationale: Diabetes mellitus places the client at risk for microvascular damage and progressive peripheral arterial disease. Twenty-five percent is within the recommended range for daily fat intake, and diet does not place the client at risk for development of peripheral arterial disease.

A patient with a cardiac disease presents with an elevated homocysteine level. Which diet instruction should the nurse recommend to the patient? A. A diet with total fat that is less than 30% of total calories. B. A diet that includes green vegetables C. Increasing fiber in the diet D. Avoiding canned food and processed meats

B. A diet that includes green vegetables Rationale: The best way to work with the folate/B-complex vitamin deficiency is to provide foods that are high in folic acid.

Which assessment finding would cause the nurse the greatest concern in a patient with hypertension? A. A patient has BP readings ranging from 120/70 to 150/85. B. A urine sample shows positive albumin C. A patient has a BP of 130/80 in the right arm and 135/85 in the left arm. D. A dietary history demonstrated that the patient is consuming processed and instant foods.

B. A urine sample shows positive albumin Rationale: This is indicative of target organ damage, specifically to the kidneys.

A nurse is caring for a client who has chronic venous insufficiency and a prescription for thigh-high compression stockings. Which of the following actions should the nurse take? A. Elevate the client's legs for 10min, two to three times daily while wearing stockings B. Apply the stockings in the morning upon awakening and before getting out of bed C. Roll the stockings down to the knees to relieve discomfort on the legs D. Remove the stockings while out of bed for 1 hr, four times a day, to allow the legs to rest

B. Apply the stockings in the morning upon awakening and before getting out of bed

A nurse is preparing to administer packed RBCs to a client who has a Hgb of 8 g/dL. Which of the following actions should the nurse plan to take during the first 15 min of the transfusion? A. Obtain consent from the client for the transfusion B. Assess for an acute hemolytic reaction C. Explain the transfusion procedure to the client D. Obtain blood culture specimens to send to the lab

B. Assess for an acute hemolytic reaction

The patient is admitted to the emergency room with suspected heart failure. Which action should the nurse take first? A. Ask the patient about his symptoms B. Assess the heart rate and lungs sounds C. Check for peripheral edema D. Check troponin and electrolyte levels

B. Assess the heart rate and lungs sounds Rationale: We want to check to make sure they are not having acute decompensated HF. Suspected HF is also an issue of the ABCs, so checking these is our priority.

A nurse is caring for a client who has heart failure and reports increased shortness of breath. Which of the following actions should the nurse take first? A. Obtain the client's weight B. Assist the client into high-Fowler's position C. Auscultate lung sounds D. Check oxygen saturation with pulse oximeter

B. Assist the client into high-Fowler's position

A nurse is assessing a client who is undergoing hemodynamic monitoring. The client has a CVP of 7 mmHg and a PAWP of 17 mmHg. Which of the following findings should the nurse expect? (Select all that apply.) A. Poor skin turgor B. Bilateral crackles in the lungs C. Jugular vein distention D. Dry mucous membranes E. Hepatomegaly

B. Bilateral crackles in the lungs C. Jugular vein distention E. Hepatomegaly

A nurse is caring for a client who is 8 hr postoperative following a coronary artery bypass graft (CABG). Which of the following findings should the nurse report? A. Mediastinal drainage 100 mL/hr B. Blood pressure 160/80 mm Hg C. Temperature 37.1° C (98.8° F) D. Potassium 4.0 mEq/L

B. Blood pressure 160/80 mm Hg Rationale: The nurse should report an elevated blood pressure following a CABG because increased vascular pressure can cause bleeding at the incision sites. Mediastinal drainage of up to 150 mL/hr is expected during this time.

The patient is returning to the floor after a cardiac catheterization. Two hours later, the patient complains of numbness and pain in the right foot. Which action should the nurse take next? A. Call physician B. Check the pedal pulse C. Take the blood pressure D. Inform the patient this is expected E. Continue to monitor

B. Check the pedal pulse Rationale: The first step is to check the pedal pulse to assess for impaired peripheral perfusion. Calling the physician is definitely a necessary step, BUT in order to inform the physician about the patient's condition, we need to assess the pedal pulse first. (Hint: if calling a physician is an option in a question and there is an assessment option associated with the patient's situation, the answer is USUALLY that assessment before calling the physician)

For a patient with a thoracentesis, which assessment would be of concern? A. Decrease in accessory muscles B. Decreased breath sounds C. Pain at the puncture site D. Increased appetite

B. Decreased breath sounds

A nurse is assessing a client who is 4 hr postoperative following a total laryngectomy. Which of the following findings is the priority for the nurse to report to the provider? A. Bleeding at the surgical site B. Decreased oxygen saturation C. Urinary retention D. Increased pain level

B. Decreased oxygen saturation Rationale: When using the airway, breathing, circulation approach to client care, the nurse should identify decreased oxygen saturation as the priority finding to address and report to the provider. A client who is postoperative following a total laryngectomy is at higher risk for hypoxia because of airway obstruction. Bleeding at the surgical site requires intervention by the nurse because hemorrhage is a complication of the procedure. However, the decreased oxygen saturation is the priority for the nurse to report to the provider.

A nurse is screening a client for hypertension. The nurse should identify that which of the following actions by the client increase the risk for hypertension? (Select all that apply.) A. Drinking 8 oz nonfat milk daily B. Eating popcorn at the movie theater C. Walking 1 mile daily at 12 min/mile pace D. Consuming 36 oz beer daily E. Getting a massage once a week

B. Eating popcorn at the movie theater D. Consuming 36 oz beer daily

Current vital signs: BP 170/100, HR 96, RR 16, chest discomfort, slight ankle edema after standing all day, smokes cigarettes Besides monitoring Carlos for decreased perfusion to the heart, which test would best confirm if heart failure is present? A. Electrocardiogram B. Echocardiogram C. Serum troponin D. Holter monitor

B. Echocardiogram

A nurse is assessing a client who has emphysema. Which of the following findings should the nurse report to the provider? A. Rhonchi on inspiration B. Elevated temperature C. Barrel-shaped chest D. Diminished breath sounds

B. Elevated temperature Rationale: The nurse should report an elevated temperature to the provider because it can indicate a possible respiratory infection. Clients who have emphysema are at risk for the development of pneumonia and other respiratory infections. Rhonchi on inspiration, barrel-shaped chest, and diminished breath sounds are expected findings in a client who has emphysema.

A nurse is assessing a client who has a history of asthma. Which of the following factors should the nurse identify as a risk for asthma? A. Sex B. Environmental allergies C. Alcohol use D. History of diabetes

B. Environmental allergies

The nurse is teaching a client about a thoracentesis. How would the nurse best describe this procedure? A. You will be asleep when they make the incision B. Fluid will be drawn off the area around your lung C. It involves lying very still in a large noisy machine D. Your doctor will be able to see inside the lung

B. Fluid will be drawn off the area around your lung

A nurse is assessing a client who has a chest tube and drainage system in place. Which of the following are expected findings? (Select all that apply.) A. Continuous bubbling in the water seal chamber B. Gentle constant bubbling in the suction control chamber C. Rise and fall in the level of water in the water seal chamber with inspiration and expiration D. Exposed sutures without dressing E. Drainage system upright at chest level

B. Gentle constant bubbling in the suction control chamber C. Rise and fall in the level of water in the water seal chamber with inspiration and expiration

Which food should the nurse recommend that a patient avoid who is taking Theophylline for COPD? A. Grapefruit juice B. Hot chocolate C. Ginger ale D. Spring water

B. Hot chocolate Rationale: Hot chocolate contains caffeine. Theophylline is also a stimulant, so the patient may be too stimulated.

The nurse understands that the priority action for a patient with a chest tube that has come disconnected from the chest-drainage system is which of the following? A. Immediately cover the end of the chest tube with a sterile dressing. B. Immediately submerge the end of the chest tube in sterile water. C. Immediately reconnect the end of the chest tube with the drainage system. D. Immediately page the provider to insert a new chest tube.

B. Immediately submerge the end of the chest tube in sterile water.

A patient with hypertension has which physical symptom? A. Decreased resistance, which may increase CO B. Increased resistance, which may decrease CO C. Increased resistance, which may increase CO D. Decreased resistance, which may decrease CO

B. Increased resistance, which may decrease CO

A nurse is planning care for a client who has severe acute respiratory distress syndrome (SARS). Which of the following actions should the nurse include? (Select all that apply.) A. Administer antibiotics B. Provide supplemental oxygen C. Administer antiviral medications D. Administer bronchodilators E. Maintain ventilators support

B. Provide supplemental oxygen D. Administer bronchodilators E. Maintain ventilators support

You are caring for a client who sustained a chest injury following a motor vehicle collision, requiring a chest tube, and is receiving morphine via patient-controlled analgesia (PCA) for pain. Which of the following would alarm you and initiate a call to the primary care provider? A. Somnolence B. Restlessness and anxiety C. Itching at the IV site D. Minimal amounts of bloody chest tube drainage

B. Restlessness and anxiety

A nurse is caring for a client who is receiving a blood transfusion and reports itching. The nurse observes areas of urticaria on the client's skin. Which of the following actions should the nurse take? A. Administer prescribed antipyretic. B. Stop the blood transfusion. C. Reevaluate the client in 15 min. D. Apply a warm compress to the affected areas.

B. Stop the blood transfusion. Rationale: This client is exhibiting manifestations of a mild allergic reaction to the blood transfusion. The nurse should stop the transfusion and follow facility protocol regarding transfusion reactions.

The nurse is managing the care of a patient with an arterial line. Which assessment finding warrants immediate intervention by the nurse? A. An overdamped waveform on the monitor B. Tubing disconnected from the arterial line C. IV medications being infused into an arterial line D. Redness at the arterial line insertion site

B. Tubing disconnected from the arterial line

A new graduate nurse is caring for a patient with a pulse ox of 89% who is experiencing dyspnea due to heart failure and COPD. Which nursing action requires intervention by the charge nurse? (Select all that apply.) A. Elevate the bed 30 degrees B. Turn and reposition the client every 1-2 hours C. Teach the client purse-lip breathing D. Weigh the client daily in the morning E. Apply oxygen at 3 liters/min

B. Turn and reposition the client every 1-2 hours E. Apply oxygen at 3 liters/min Rationale: If my patient has HF and COPD, we want them to rest as much as possible and conserve their energy and oxygen. Repositioning the client will require too much of their energy and decrease their already diminished oxygen supply. 3 L of oxygen per minute is too much, we want to administer no more than 2 L/min.

A nurse is monitoring a client's ECG monitor and notes the client's rhythm has changed from normal sinus rhythm to supraventricular tachycardia. The nurse should prepare to assist with which of the following interventions? A. Initiate chest compressions B. Vagal stimulation C. Administration of atropine IV D. Defibrillation

B. Vagal stimulation Rationale: The nurse should identify that vagal stimulation might temporarily convert the client's heart rate to normal sinus rhythm. The nurse should have a defibrillator and resuscitation equipment at the client's bedside because vagal stimulation can cause bradydysrhythmias, ventricular dysrhythmias, or asystole.

The nurse is consulting with a nutritionist about the dietary therapy for a patient with chronic venous stasis ulcers. What are the dietary recommendations would be most important to recommend to this patient? A. Low sodium foods B. Vitamin A, & C and zinc C. Low fat foods D. High calcium foods

B. Vitamin A, & C and zinc Rationale: The most important thing we want this ulcer to do is HEAL! Vitamin A and C and zinc will help promote building collagen and promote wound healing.

A nurse is providing discharge teaching to a client who has a temporary tracheostomy. Which of the following statements by the client indicates an understanding of the teaching? A. "I should dip a cotton-tipped applicator into full-strength hydrogen peroxide to cleanse around my stoma." B. "I should cut a 4-inch gauze dressing and place it around my tracheostomy tube to absorb drainage." C. "I should remove the old twill ties after the new ties are in place." D. "I should apply suction while inserting the catheter into my tracheostomy tube."

C. "I should remove the old twill ties after the new ties are in place." Rationale: As a safety measure, the nurse should teach the client to wait until the new ties are in place to remove the old ties. This practice can prevent accidental decannulation. The client should use gauze squares moistened in 0.9% sodium chloride to cleanse around the stoma or, if prescribed, half-strength hydrogen peroxide can be used on the skin to clean crusty areas. Using a cotton-tipped applicator places the client at risk for aspiration of cotton fibers. Also, the client should be careful not to get hydrogen peroxide into the tracheal stoma. Cutting a 4-inch square gauze dressing places the client at risk for aspiration of gauze fibers. The client should apply a commercially-prepared split gauze tracheostomy dressing under the flange of the tracheostomy tube. The client should apply suction only when withdrawing the catheter to prevent tracheal tissue trauma.

Which statement by the patient about the need for antihypertensive medication indicates the need for further teaching? A. "I'm worried about a stroke if my BP is not controlled." B. "Can my kidneys fail if I don't control my BP?" C. "My BP is only slightly elevated, so I am ok." D. "I guess I need to take this medication even if I feel OK."

C. "My BP is only slightly elevated, so I am ok."

A nurse is planning to instruct a client on how to perform pursed-lip breathing. Which of the following statements should the nurse include? A. "Take quick breaths upon inhalation." B. "Place your hand over your stomach." C. "Take a deep breath through your nose." D. "Puff your cheeks upon exhalation."

C. "Take a deep breath through your nose."

A newly licensed nurse is observing a cardioversion procedure and hears the team leader call out, "Stand clear." This statement indicates which of the following events is occurring? A. "The cardioverter is being charged to the appropriate setting." B. "The team should initiate CPR due to pulseless electrical activity." C. "Team members cannot be in contact with the equipment connected to the client." D. "A time-out is being called to verify correct protocols."

C. "Team members cannot be in contact with the equipment connected to the client."

A nurse is talking with a client who has class I heart failure and asks about obtaining a ventricular assist device (VAD). Which of the following statements should the nurse make? A. "VADs are only implanted during heart transplantation." B. "A VAD helps to pace the heart." C. "VADs are used when heart failure is not responsive to medications." D. "A VAD is useful for clients who also have a chronic lung issue."

C. "VADs are used when heart failure is not responsive to medications."

A nurse is caring for a client who has a new diagnosis of tuberculosis and has been placed on a multimedication regimen. Which of the following instruction should the nurse give the client related to ethambutol? A. "Your urine can turn a dark orange." B. "Watch for a change in the sclera of your eye." C. "Watch for any changes in vision." D. "Take vitamin B6 daily."

C. "Watch for any changes in vision." Rationale: Optic neuritis can result from taking this medication.

A nurse is providing preoperative teaching for a client who requests autologous donation in preparation for a scheduled orthopedic surgical procedure. Which of the following statements should the nurse include in the teaching? A. "You should make an appointment to donate blood 8 weeks prior to the surgery." B. "If you need an autologous transfusion, the blood your brother donates can be used." C. "You can donate blood each week if your hemoglobin is stable." D. "Any unused blood that is donated can be used for other clients."

C. "You can donate blood each week if your hemoglobin is stable."

A nurse is preparing to administer a new prescription for isoniazid (INH) to a light-skinned client who has tuberculosis. The nurse should instruct the client to report which of the following findings as an adverse effect of the medication? A. "You might notice yellowing of your skin." B. "You might experience pain in your joints." C. "You might notice tingling in your hands." D. "You might experience a loss of appetite."

C. "You might notice tingling in your hands."

The nurse determines which patient is at greatest risk for developing infective endocarditis (IE)? A. A 22-year-old student undergoing a dental procedure B. A 35-year-old man with a past medical history of IV drug use C. A 65-year-old male heart transplant patient on immunosuppressive therapy undergoing a colonoscopy D. A 70-year-old female with heart failure with an intravascular access device for home infusion

C. A 65-year-old male heart transplant patient on immunosuppressive therapy undergoing a colonoscopy

Carlos has the following lipid levels... HDL: 32 mg/dL LDL: 190 mg/dL Total Cholesterol: 250mg/dL Carlos inquires about these levels. Which statement is most appropriate by the nurse? A. Total cholesterol below 300mg/dL is normal B. As long as the LDL is below 200 mg/dL no medications are needed C. A HDL level below 40 mg/dL is a risk factor for heart disease D. These levels are borderline and given your risk factors, diet and activity changes need to be made.

C. A HDL level below 40 mg/dL is a risk factor for heart disease Rationale: Total cholesterol should be under 200 mg/dL and LDL under 100 mg/dL. HDL should be between 40-60 mg/dL.

A nurse in the ED is assessing a client who was in a motor vehicle crash. Findings include absent breath sounds in the left lower lobe with dyspnea, BP 118/68, HR 124, RR 38, temp 38.6C (101.4F), and SaO2 92% on room air. Which of the following actions should the nurse take first? A. Obtain a chest x-ray B. Prepare for chest tube insertion C. Administer oxygen via a high-flow mask D. Initiate IV access

C. Administer oxygen via a high-flow mask

A nurse is caring for a client who is in acute respiratory failure and is receiving mechanical ventilation. Which of the following assessments is the best method for the nurse to use to determine the effectiveness of the current treatment regimen? A. Blood pressure B. Capillary refill C. Arterial blood gases D. Heart rate

C. Arterial blood gases Rationale: When using the airway, breathing, circulation approach to client care, the nurse should place priority on evaluating arterial blood gases to determine serum oxygen saturation and acid-base balance.

A nurse is caring for a client who is receiving mechanical ventilation when the low-pressure alarm sounds. Which of the following situations should the nurse recognize as a possible cause of the alarm? A. Excess secretions B. Kinks in the tubing C. Artificial airway cuff leak D. Biting on the endotracheal tube

C. Artificial airway cuff leak Rationale: All other answer choices are things that would cause the high-pressure alarm to sound.

Which assessment is essential for the nurse to complete immediately following a transesophageal echocardiogram (TEE)? A. Check pedal pulses B. Monitor urine output C. Check gag reflex D. Complete a neuro assessment

C. Check gag reflex

A patient has been admitted to the hospital for a PE. What is the priority nursing intervention? A. Insert an IV line B. Begin heparin drip as ordered C. Check oxygen saturation D. Determine patient allergies

C. Check oxygen saturation

A nurse is reviewing the laboratory results of several male clients who have peripheral arterial disease. The nurse should plan to provide dietary teaching for the client who has which of the following laboratory values? A. Cholesterol 180 mg/dL, HDL 70 mg/dL, LDL 90 mg/dL B. Cholesterol 185 mg/dL, HDL 50 mg/dL, LDL 120 mg/dL C. Cholesterol 190 mg/dL, HDL 25 mg/dL, LDL 160 mg/dL D. Cholesterol 195 mg/dL, HDL 55 mg/dL, LDL 125 mg/dL

C. Cholesterol 190 mg/dL, HDL 25 mg/dL, LDL 160 mg/dL Rationale: These laboratory values for HDL and LDL are outside of the expected reference range and indicate that the nurse should provide dietary teaching to the client. The expected reference range for cholesterol is less than 200 mg/dL; for HDL is above 45 mg/dL for males and above 55 mg/dL for females; and for LDL is less than 130 mg/dL.

What is the primary difference between emphysema and chronic bronchitis? A. Chronic bronchitis predominately affects the large airways. B. Emphysema predominately affects the large airways. C. Emphysema predominately affects the alveoli. D. Chronic bronchitis predominately affects the alveoli.

C. Emphysema predominately affects the alveoli.

A nurse is monitoring a client who began receiving a unit of packed RBCs 10 min ago. Which of the following findings should the nurse identify as an indication of a febrile transfusion reaction? (Select all that apply.) A. Temperature change from 37C (98.6F) pretransfusion to 37.2C (99.0F) B. Current BP 178/90 mm Hg C. HR change from 88 bpm pretransfusion to 120 bpm D. Client report of itching E. Client appears flushed

C. HR change from 88 bpm pretransfusion to 120 bpm E. Client appears flushed

When completing a history and physical examination on a patient with a hematological disorder, which action is appropriate? A. Deeply palpate the spleen to determine the extent of splenomegaly B. Perform a respiratory assessment after moderate exercise for accurate measurement of depth and rhythm C. Inspect oral mucous membranes and the tongue for lesions, swelling, and pain D. Suggest a bone marrow biopsy if the patient reports decreased energy levels

C. Inspect oral mucous membranes and the tongue for lesions, swelling, and pain

When he first came to the office, Carlos' main complaint was chest discomfort and dyspnea. He also reported he has started cigarette smoking again and notices slight edema in his ankles after standing all day. After testing, Carlos was diagnosed with heart failure. Which finding indicates to the nurse that treatment plan for Carlos's heart failure is effective? A. Peripheral pulse has changed from +3 to +4 B. Lungs are clear to auscultation C. Minimal jugular vein distention D. Carlos reports completing ADLs without dyspnea

D. Carlos reports completing ADLs without dyspnea

A nurse is caring for a client who is 1 hr postoperative following a thoracentesis. Which of the following is the priority assessment finding? A. Pallor B. Insertion site pain C. Persistent cough D. Temperature 37.3° C (99.1° F)

C. Persistent cough Rationale: When using the airway, breathing, circulation approach to client care, the nurse should determine that the priority finding is a persistent cough because this can indicate a tension pneumothorax, which is a medical emergency. Pallor is an important finding because it can indicate blood loss. Insertion site pain is an important finding because untreated pain can result in shallow respirations. An elevated temperature is an important finding because it can indicate infection. However, these assessment findings are not as important as the persistent cough.

A nurse is creating a plan of care for a client who has COPD. Which of the following interventions should the nurse include? A. Schedule respiratory treatments following meals. B. Have the client sit up in a chair for 2-hr periods three times per day. C. Provide a diet that is high in calories and protein. D. Combine activities to allow for longer rest periods between activities.

C. Provide a diet that is high in calories and protein. Rationale: The nurse should provide a client who has COPD with a diet that is high in calories and protein and low in carbohydrates.

Which activity should be avoided in a patient with sickle cell anemia? A. Driving to the beach 3 hours away B. Going to a concert C. Running in a 5-k race D. Carpentry work

C. Running in a 5-k race

Which factor places the patient at a higher risk for developing secondary polycythemia? A. Type 2 diabetes mellitus B. History of alcohol abuse C. Smoking D. Hypertension

C. Smoking

A nurse is presenting a community education program on recommended lifestyle changes to prevent angina and MI. Which of the following changes should the nurse recommend be made first? A. Diet modification B. Relaxation exercises C. Smoking cessation D. Taking omega-3 capsules

C. Smoking cessation

A nurse in an urgent care clinic is obtaining a history from a client who has type 2 diabetes mellitus and a recent diagnosis of hypertension. This is the second time in 2 weeks that the client has experienced hypoglycemia. Which of the following client data should the nurse report to the provider? A. Takes psyllium daily as a fiber laxative B. Drinks skin milk daily as a bedtime snack C. Takes metoprolol daily after meals D. Drinks grapefruit juice daily with breakfast

C. Takes metoprolol daily after meals

The correct statement about the prevalence of pneumonia is which of the following? A. Pneumonia is no longer a major health problem. B. The prevalence of pneumonia is increasing dramatically. C. The prevalence and mortality are highest in persons older than 65 years. D. The frequency has not declined, but mortality has declined.

C. The prevalence and mortality are highest in persons older than 65 years.

A nurse on a cardiac unit is reviewing the laboratory findings of a client who has a diagnosis of myocardial infarction (MI) and reports that his dyspnea began 2 weeks ago. Which of the following cardiac enzymes would confirm the MI occurred 14 days ago? A. CK-MB B. Troponin I C. Troponin T D. Myoglobin

C. Troponin T

A nurse is caring for a client who is 1 hr postoperative following an aortic aneurysm repair. Which of the following findings can indicate shock and should be reported to the provider? A. Serosanguineous drainage on dressing B. Severe pain with coughing C. Urine output of 20 mL/hr D. Increase in temperature from 36.8° C (98.2° F) to 37.5° C (99.5° F)

C. Urine output of 20 mL/hr Rationale: Urine output less than 30 mL/hr is a manifestation of shock. Urine output is decreased due to a compensatory decreased blood flow to the kidneys, hypovolemia, or graft thrombosis or rupture. Serosanguineous drainage 1 hr postoperative is expected and is not a manifestation of shock. Serosanguineous drainage should decrease over the first few days and discontinue after day 5.

A client with asthma has pronounced wheezing upon auscultation. Which action should the nurse take? A. Have the client cough and deep breathe B. Prepare to intubate the client C. Notify the health care provider D. Administer a nebulized beta 2 agonist

D. Administer a nebulized beta 2 agonist Rationale: Beta-2 agonists are our rescue meds and will function to open up the airways. We need to open up the airways ASAP to restore normal breathing.

A nurse in an emergency department is caring for a client who had an anterior MI. The client's history reveals they are 1 week postoperative following an open cholecystectomy. The nurse should identify that which of the following interventions is contraindicated? A. Administering IV morphine sulfate B. Administering oxygen at 2 L/min via nasal cannula C. Helping the client to the bedside commode D. Assisting with thrombolytic therapy

D. Assisting with thrombolytic therapy Rationale: The nurse should recognize that major surgery within the previous 3 weeks is a contraindication for thrombolytic therapy. The nurse should administer IV morphine to the client to relieve pain and reduce myocardial oxygen demand.

A nurse is assessing a client following a bronchoscopy. Which of the following findings should the nurse report to the provider? A. Blood-tinged sputum B. Dry, nonproductive cough C. Sore throat D. Bronchospasms

D. Bronchospasms

The patient has a peripheral arterial bypass with graft placement. The nurse notes erythema, tenderness and drainage at the site. What should the nurse do next? A. Notify the physician B. Auscultate the patient's lungs and check the pulse oximeter C. Assess the patient for signs of bleeding and reviews the PT results D. Checks the patient's temperature and looks at the white blood cell count.

D. Checks the patient's temperature and looks at the white blood cell count. Rationale: Erythema, tenderness, and drainage are signs of possible infection.

A nurse started a transfusion of packed RBCs for a client 1 hr ago. The client has suddenly developed shaking chills, muscle stiffness, and a temperature of 38.6C (101.5F). The client appears and reports a headache and "nervousness". The nurse should identify that the client has most likely developed which of the following types of transfusion reaction? A. Septic B. Acute hemolytic C. Allergic D. Febrile nonhemolytic

D. Febrile nonhemolytic Rationale: This is the most common type of transfusion reaction. The characteristic fever usually develops within 2 hr after the transfusion is started. Other classic symptoms include chills, headache, flushing, anxiety, and muscle pain. This type of reaction is usually a result of sensitization to the plasma, platelets, or white blood cells. Although this type of reaction is not life-threatening, it can be frightening and uncomfortable for the client.

A nurse in a clinic is caring for a client who has suspected anemia. Which of the following laboratory test results should the nurse expect? A. Iron 90mcg/dL B. RBC 6.5 million/uL C. WBC 4,800 mm3 D. Hgb 10 g/dL

D. Hgb 10 g/dL

The nurse is admitting an older adult with decompensated congestive heart failure. The nursing assessment reveals adventitious lung sounds, dyspnea, and orthopnea. The nurse should question which doctor's order? A. Furosemide (Lasix) 20 mg po now B. Oxygen via face mask at 2 L C. KCl 20 mEq PO two times per day D. Intravenous (IV) 500 mL of 0.9% NaCl at 125 mL/hr

D. Intravenous (IV) 500 mL of 0.9% NaCl at 125 mL/hr Rationale: The patient is already experiencing fluid overload, so we don't want to give the patient more fluid and further exacerbate their heart failure.

Carlos is instructed to report to the cardiology center the next day for a stress echo test. Which instruction should the nurse provide to Carlos? A. The dye that is inserted may cause a warm flushing sensation. B. It is important to keep a record of activities while you are wearing the monitor C. You will need to be NPO after midnight D. It is important to return to the table immediately following exercising.

D. It is important to return to the table immediately following exercising.

Carlos is found to have mild heart failure and is treated with low dose Digoxin and Lasix. To prevent possible complications of this drug combination, the nurse should take which action? A. Keep an accurate measure of intake and output B. Teach the patient about dietary restriction of sodium C. Take the apical pulse and withhold the heart medication if the heart rate is irregular D. Monitor the serum potassium levels

D. Monitor the serum potassium levels

A patient with lymphoma is beginning the induction chemotherapy regimen. Which information is most essential for the nurse to include in the treatment plan? A. Advance directives B. Bleeding precautions C. Importance of frequent rest periods D. Neutropenic precautions

D. Neutropenic precautions

A nurse preceptor is observing a newly licensed nurse on the unit who is preparing to administer a blood transfusion to an older adult client. Which of the following actions by the newly licensed nurse indicates understanding of the procedure? A. Inserts an 18-gauge IV catheter in the client B. Verifies blood compatibility and expiration date of the blood with an assistive personnel C. Administers dextrose 5% in 0.9% sodium chloride IV with the transfusion D. Obtains vital signs every 15 min throughout the procedure

D. Obtains vital signs every 15 min throughout the procedure

Which of the following is a finding attributable to age-related changes? A. Kyphosis B. AP-to-lateral diameter of 1:1 C. PaCO2 of 47 on an arterial blood gas D. PaCO2 of 60 on an arterial blood gas

D. PaCO2 of 60 on an arterial blood gas

A nurse in an ED is caring for a client who is experiencing acute respiratory failure. Which of the following laboratory findings should the nurse expect? A. Arterial pH 7.50 B. PaCO2 25 mm Hg C. SaO2 92% D. PaO2 58 mm Hg

D. PaO2 58 mm Hg Rationale: The nurse should expect the client to have lower partial pressures of oxygen. The nurse should expect the client to have a decrease in oxygen saturation, an increase in carbon dioxide level, and a decreased pH because respiratory failure can cause respiratory acidosis.

A nurse is assisting a provider with the removal of a chest tube. Which of the following actions should the nurse take? A. Instruct the client to lie prone with arms by the sides B. Complete a surgical checklist on the client C. Remind the client that there is minimal discomfort during the removal process D. Place an occlusive dressing over the site once the tube is removed

D. Place an occlusive dressing over the site once the tube is removed

A nurse is teaching a client who has a new diagnosis of severe peripheral arterial disease. Which of the following instructions should the nurse include? A. Wear tightly-fitted insulated socks with shoes when going outside B. Elevate both legs above the heart when resting C. Apply a heating pad to both legs for comfort D. Place both legs in dependent position while sleeping

D. Place both legs in dependent position while sleeping

A nurse is caring for a client who is receiving heparin therapy and develops hematuria. Which of the following actions should the nurse take if the client's aPTT is 96 seconds? A. Increase the heparin infusion flow rate by 2 mL/hr. B. Continue to monitor the heparin infusion as prescribed. C. Request a prothrombin time (PT). D. Stop the heparin infusion.

D. Stop the heparin infusion. Rationale: The nurse should identify that the client's aPTT is above the critical value and the client is displaying manifestations of bleeding. Therefore, the nurse should discontinue the heparin infusion immediately and notify the provider to reduce the risk of client injury.

What is the role of erythropoietin in the regulation of red blood cells? A. To make Hgb capable of transporting oxygen B. To pick up carbon dioxide from the tissues and deliver it to the lungs C. To decrease RBC production D. To stimulate RBC production

D. To stimulate RBC production

A nurse working in an ED is caring for a client following acute chest trauma. Which of the following findings should indicate to the nurse that the client is possibly experiencing a tension pneumothorax? A. Collapsed neck veins on the affected side B. Collapsed neck veins on the unaffected side C. Tracheal deviation to the affected side D. Tracheal deviation to the unaffected side

D. Tracheal deviation to the unaffected side Rationale: The nurse should recognize that deviation of the trachea to the unaffected side is a possible indicator that the client is experiencing a tension pneumothorax. A tension pneumothorax results from free air filling the chest cavity, causing the lung to collapse and forcing the trachea to deviate to the unaffected side.


Kaugnay na mga set ng pag-aaral

STANDARD COSTING AND VARIANCE ANALYSIS

View Set

CompTIA Security+ Cert. (SY0-501): Practice Tests #3

View Set

Applications and Underwriting chapter 10

View Set